0% found this document useful (1 vote)
14K views140 pages

AP Physics 1 Practice Exam #2 For The 2022 Exam

Uploaded by

liubingvera
Copyright
© © All Rights Reserved
We take content rights seriously. If you suspect this is your content, claim it here.
Available Formats
Download as PDF, TXT or read online on Scribd
0% found this document useful (1 vote)
14K views140 pages

AP Physics 1 Practice Exam #2 For The 2022 Exam

Uploaded by

liubingvera
Copyright
© © All Rights Reserved
We take content rights seriously. If you suspect this is your content, claim it here.
Available Formats
Download as PDF, TXT or read online on Scribd
You are on page 1/ 140

AP Physics 1: Algebra-Based

Practice Exam #2
For the
Spring 2022
Exam
Please note: Some multiple-choice and free-response questions previously
included in this practice exam have been removed because the content of the
questions is no longer within the scope of the AP Physics 1 course. Additional
questions have been added to create a practice exam that has the appropriate
number of items in each section. Because this is a practice exam and has not
been administered, the scoring worksheet found on p 138 provides only
approximate AP Exam scores. Student performance on this practice exam does
not necessarily predict performance on a different exam.

Additional information about the AP Physics 1 Course and Exam can be found
here:

https://apcentral.collegeboard.org/courses/ap-physics-1

This exam may not be posted on school or personal websites, nor


electronically redistributed for any reason. This exam is provided by
College Board for AP Exam preparation. Teachers are permitted to download
the materials and make copies to use with their students in a classroom
setting only. To maintain the security of this exam, teachers should collect all
materials after their administration and keep them in a secure location.

Further distribution of these materials outside of the secure College


Board site disadvantages teachers who rely on uncirculated questions
for classroom testing. Any additional distribution is in violation of College
Board’s copyright policies and may result in the termination of Practice Exam
access for your school as well as the removal of access to other online
services such as the AP Teacher Community and Online Score Reports.

© 2022 College Board. College Board, Advanced Placement, AP, AP Central, and the acorn logo are registered
trademarks of College Board.
Visit College Board on the web: collegeboard.org.
Contents

Exam Instructions

Student Answer Sheet for the Multiple-Choice Section

Section I: Multiple-Choice Questions

Section II: Free-Response Questions

Multiple-Choice Answer Key

Course Framework Alignment and Rationales

Free-Response Scoring Guidelines

Scoring Worksheet

Question Descriptors and Performance Data

Note: This publication shows the page numbers that appeared in


the 2020−21 AP Exam Instructions book and in the actual exam.
This publication was not repaginated to begin with page 1.

© 2022 College Board. College Board, Advanced Placement Program, AP, SAT and the acorn logo
are registered trademarks of College Board. All other products and services may be trademarks of their
respective owners. Permission to use copyrighted College Board materials may be requested online at:
www.collegeboard.org/request-form.
AP Coordinators and Proctors:

Update: Managing the Exam Break


In the 2020-21 AP Exam Instructions, you’ll As per standard AP policy, the break must be
see information about the standard 10-minute monitored. Please note that the decision to
break between Sections I and II of the paper- lengthen the break is optional, does not need
and-pencil AP Exams. to be applied to every testing room uniformly,
and is not required to administer AP Exams.
This year, local health and safety guidelines If your school chooses to provide a longer
may require that only a limited number of break, you’re not required to use all the
students may access the restroom or additional time.
hallways at the same time. In such cases, you
may elect to double the time of the scheduled
breaks (including accommodated breaks) to
ensure that students can access the restroom
or hallway in an orderly fashion.

© 2021 College Board. Visit College Board on the web: collegeboard.org.


AP Physics 1:
Algebra-Based Exam
Administration 1 Exam Date: Wednesday afternoon, May 5, 2021
Administration 2 Exam Date: Monday afternoon, May 24, 2021

AP Physics 2:
Algebra-Based Exam
Administration 1 Exam Date: Friday afternoon, May 7, 2021
Administration 2 Exam Date: Tuesday afternoon, May 25, 2021

Section I Total Time: 1 hour and 30 minutes


Calculator allowed
Number of Questions: 50
Percent of Total Score: 50%
Writing Instrument: Pencil required

Section II Total Time: 1 hour and 30 minutes


Calculator allowed
Number of Questions Physics 1: 5
Number of Questions Physics 2: 4
Percent of Total Score: 50%
Writing Instrument: Pen with black or dark blue ink, or pencil

What Proctors Need to Bring to This Exam


Exam packets Container for students’ electronic devices (if needed)
Answer sheets Extra No. 2 pencils with erasers
The personalized AP ID label sheet for each Extra pens with black or dark blue ink
student taking the exam Unlined paper
Part 2 of the 2020-21 AP Coordinator’s Manual Stapler
This book—2020-21 AP Exam Instructions Watch
AP Exam Seating Chart template Signs for the door to the testing room
Extra calculators –– “Exam in Progress”
Extra rulers or straightedges – “Phones of any kind are prohibited during the
Pencil sharpener exam administration, including breaks”

AP Physics 1 and Physics 2 Exams  205


© 2021 College Board. Visit College Board on the web: collegeboard.org.
2020-21 AP Exam Instructions

NOTE: A four-function, scientific, or graphing calculator may be used on all sections of the
AP Physics 1: Algebra-Based and Physics 2: Algebra-Based Exams. See “Calculator Policy”
in Part 2 of the 2020-21 AP Coordinator’s Manual for details about the calculator policy.

Before Distributing Exams: Check that the title on all exam covers is
Physics 1: Algebra-Based or Physics 2: Algebra-Based. If there are any exam booklets
with a different title, contact the AP coordinator immediately.

Calculator Use
Students are permitted to use rulers, straightedges, and four-function, scientific, or
graphing calculators for these entire exams (Sections I and II). Before starting the exam
administration, make sure each student has an appropriate calculator, and any student with
a graphing calculator has a model from the approved list on the “AP-Approved Graphing
Calculators” table in Part 2 of the 2020-21 AP Coordinator’s Manual. If a student does not
have an appropriate calculator or has a graphing calculator not on the approved list, you
may provide one from your supply. If the student does not want to use the calculator you
provide or does not want to use a calculator at all, they must hand copy, date, and sign the
“Calculator Release Statement” in Part 2 of the 2020-21 AP Coordinator’s Manual.

Students may have no more than two calculators on their desks. Calculators may not be
shared. Calculator memories do not need to be cleared before or after the exam. Students
with Hewlett-Packard 48–50 Series and Casio FX-9860 graphing calculators may use cards
designed for use with these calculators. Proctors should make sure infrared ports (Hewlett-
Packard) are not facing each other. Since graphing calculators can be used to store
data, including text, proctors should monitor that students are using their calculators
appropriately. Attempts by students to use the calculator to remove exam questions
and/or answers from the room may result in the cancellation of AP Exam scores.

Tables containing equations commonly used in physics are included in the Section I
Multiple Choice booklet and the Section II orange Reference booklet. The equation tables
are provided for use during the entire exam. Students are not allowed to bring their own
copies of the equation tables to the exam room.

SECTION I: Multiple Choice


Before starting the exam administration, make sure each student has an appropriate
calculator for the exam. See details in the section above.
› Do not begin the exam instructions below until you have completed the
General Instructions.

Make sure you administer the correct exam on the scheduled date and begin the exam
at the designated time. Remember, you must complete a seating chart for this exam. See
pages 273–274 for a seating chart template and instructions. See “Seating Policy” in Part 2
of the 2020-21 AP Coordinator’s Manual for exam seating requirements.

206 AP Physics 1 and Physics 2 Exams


© 2021 College Board. Visit College Board on the web: collegeboard.org.
2020-21 AP Exam Instructions

AP Physics 1 and Physics 2 Exams


Physics 1: Algebra-Based
If you are giving the exam during Administration 1, say:
It is Wednesday afternoon, May 5, and you will be taking the AP Physics 1:
Algebra-Based Exam. Look at your exam packet and confirm that the exam title
is “AP Physics 1: Algebra-Based.” Raise your hand if your exam packet contains
any other exam title and I will help you.

If you are giving the exam during Administration 2, say:


It is Monday afternoon, May 24, and you will be taking the AP Physics 1:
Algebra-Based Exam. Look at your exam packet and confirm that the exam title
is “AP Physics 1: Algebra-Based.” Raise your hand if your exam packet contains
any other exam title and I will help you.

Physics 2: Algebra-Based
If you are giving the exam during Administration 1, say:
It is Friday afternoon, May 7, and you will be taking the AP Physics 2:
Algebra-Based Exam. Look at your exam packet and confirm that the exam title
is “AP Physics 2: Algebra-Based.” Raise your hand if your exam packet contains
any other exam title and I will help you.

If you are giving the exam during Administration 2, say:


It is Tuesday afternoon, May 25, and you will be taking the AP Physics 2:
Algebra-Based Exam. Look at your exam packet and confirm that the exam title
is “AP Physics 2: Algebra-Based.” Raise your hand if your exam packet contains
any other exam title and I will help you.

Once you confirm that all students have the correct exam, say:
In a moment, you will open the exam packet. By opening this packet, you agree
to all of the AP Program’s policies and procedures referenced in the AP Exam
Terms and Conditions.
You may now remove the shrinkwrap from the outside only of your exam packet.
Do not open the Section I booklet; do not remove the shrinkwrap from the
Section II booklets. Put the white seals and the shrinkwrapped Section II booklet
booklets aside. . . .
Carefully remove the AP Exam label found near the top left of your exam
booklet cover. Place it on the front of your answer sheet on the light blue box
near the bottom that reads “AP Exam Label.”. . .
If students accidentally place the exam label in the space for the AP ID label or vice versa,
advise them to leave the labels in place. They should not try to remove the label; their exam
can still be processed correctly.
Now turn over your answer sheet to the side marked page 2 and look at Item I
at the top of the page. Print the name of this exam, the exam form, and the
form code.
The exam form and the form code are located on the front cover of the
Section I booklet in the lower right corner; ignore the large number under these
two items—it is not part of the form or form code.
Look up when you have finished Item I. . . .

AP Physics 1 and Physics 2 Exams  207


© 2021 College Board. Visit College Board on the web: collegeboard.org.
2020-21 AP Exam Instructions

When students have completed Item I, say:


Listen carefully to all my instructions. I will give you time to complete each
step. Please look up after completing each step. Raise your hand if you have
any questions.
Give students enough time to complete each step. Don’t move on until all students are ready.
Read the statements on the front cover of the Section I booklet. . . .
Sign your name and write today’s date. . . .
Now print your full legal name where indicated. . . .
Does anyone have any questions? . . .
Turn to the back cover of your exam booklet and read it completely. . . .
Give students a few minutes to read the entire back cover.
Are there any questions? . . .
You will now take the multiple-choice portion of the exam. You should have in
front of you the Section I: multiple-choice booklet and your answer sheet. You
may never discuss the multiple-choice exam content at any time in any form with
anyone, including your teacher and other students. If you disclose the multiple-
choice exam content through any means, your AP Exam score will be canceled.
You must complete the answer sheet using a No. 2 pencil only. Mark all of your
responses on page 2 of your answer sheet. Remember, for numbers 1 through
45, mark only the single best answer to each question. For numbers 131
through 135, mark the two best answer choices for each question. The answer
sheet has circles marked A–E for each of these questions. For this exam, you
will use only the circles marked A–D. Completely fill in the circles. If you need
to erase, do so carefully and completely. No credit will be given for anything
written in the exam booklet. Scratch paper is not allowed, but you may use the
margins or any blank space in the exam booklet for scratch work.
Your score on the multiple-choice section will be based solely on the number of
questions answered correctly.
Rulers, straightedges, and calculators may be used for the entire exam. You may
place these items on your desk.
Are there any questions? . . .
You have 1 hour and 30 minutes for this section. Once final time is called for
Section I, stop working immediately.
Open your Section I booklet and begin.

Note Start Time . Note Stop Time .

Check that students are marking their answers in pencil on their answer sheets and
that they have not opened their shrinkwrapped Section II booklets. Make sure they’ve
placed their AP ID label sheets under their chairs. Make sure that students are using their
calculators appropriately. You should also make sure that Hewlett-Packard calculators’
infrared ports are not facing each other and that students are not sharing calculators.

After 1 hour and 20 minutes, say:


There are 10 minutes remaining.

After 10 minutes, say:


Stop working. Close your exam booklet and put your answer sheet faceup on
your desk. Make sure your AP ID label and AP Exam label are on your answer
sheet. Sit quietly while I collect your answer sheets.

208 AP Physics 1 and Physics 2 Exams


© 2021 College Board. Visit College Board on the web: collegeboard.org.
2020-21 AP Exam Instructions

Collect an answer sheet from each student. Check that each answer sheet has an AP ID

AP Physics 1 and Physics 2 Exams


label and an AP Exam label.

After all answer sheets have been collected, say:


Now you must seal your exam booklet using the white seals you set aside
earlier. Affix one white seal to each area of your exam booklet cover marked
“PLACE SEAL HERE.” Fold each seal over the back cover. When you have
finished, place the booklet faceup on your desk. I will now collect your Section I
booklet. . . .
Collect a Section I booklet from each student. Check that each student has signed the front
cover of the sealed Section I booklet.
There is a 10-minute break between Sections I and II.

When all Section I materials have been collected and accounted for and you are
ready for the break, say:
Please listen carefully to these instructions before we take a 10-minute break.
Get your AP ID label sheet from under your chair and put it on your desk. You
must leave your shrinkwrapped Section II packet and your AP ID label sheet
on your desk during the break. Please put all of your calculators under your
chair. Your calculators and all other items you placed under your chair at the
beginning of this exam must stay there. You are not permitted to open or access
them in any way. You are not allowed to consult teachers, other students,
notes, textbooks, or any other resources during the break. You may not make
phone calls, send text messages, use your calculators, check email, use a
social networking site, or access any electronic or communication device.
You may not leave the designated break area. Remember, you may never
discuss the multiple-choice exam content with anyone, and if you disclose the
content through any means, your AP Exam score will be canceled. Are there
any questions? . . .
You may begin your break. Testing will resume at .

Make sure students understand where the designated break area is.

IMPORTANT: Both the exam room and students in the designated break area must be
monitored.

At the start of the break, walk around the room to ensure all Section II books are accounted
for on students’ desks. Immediately contact the Office of Testing Integrity (OTI) if any book
is missing. Testing must not resume until the book is located or OTI is contacted. (See
contact information for OTI on the inside front cover.)

SECTION II: Free Response


After the break, say:
May I have everyone’s attention? Please look at your AP ID label sheet and
double-check that your name is printed at the top to ensure you’ve returned to
the correct seat. . . .

Confirm that students have their AP ID label sheet, then say:


For this section of the exam you may use a pen with black or dark blue ink or a
No. 2 pencil to write your responses. Does everyone have a pen or pencil? . . .
You may now remove the shrinkwrap from the Section II packet, but do not open
any booklets until you are told to do so. . . .

AP Physics 1 and Physics 2 Exams  209


© 2021 College Board. Visit College Board on the web: collegeboard.org.
2020-21 AP Exam Instructions

You should now have in front of you:


ƒƒ your AP ID label sheet,
ƒ the orange Section II: Free Response, Reference booklet, and
ƒ the Section II: Free Response booklet with a shaded block of information
on the cover. This booklet is where you’ll write your responses.
First, look at the front cover of the free-response booklet. Read the bulleted
statements. Look up when you have finished. . . .
Read the last statement. . . .
Print the first, middle, and last initials of your legal name in the boxes and
print today’s date where indicated. This constitutes your signature and your
agreement to the statements on the front cover. . . .
Now take an AP ID label from your label sheet and place it on the shaded box
marked “AP ID Label” at the bottom of your free-response booklet. If you
don’t have any AP ID labels, write your AP ID in the box. Look up when you
have finished. . . .
Now turn to the back cover of your free-response booklet and complete Items 1
through 3 under “Important Identification Information.” For Item 3, your school
code is printed at the top right of your AP ID label sheet. . . .
Give students time to complete Items 1 through 3.

Read Item 4. . . .
Are there any questions? . . .
Now I will collect your AP ID label sheet. Leave your label sheet on your desk.
While I collect them, read the remaining information on the back cover of your
free-response booklet. Do not open any booklets until you are told to do so.
Look up when you have finished. . . .
At this point, collect the AP ID label sheet from every student.

When you’ve finished collecting AP ID label sheets, say:


Are there any questions? . . .
Read the information on the front cover of the orange Reference booklet. Look
up when you have finished. . . .
Rulers, straightedges, and calculators may be used for Section II. Be sure these
items are on your desk. . . .
You have 1 hour and 30 minutes to complete Section II. You are responsible for
pacing yourself, and you may proceed freely from one question to the next.

If you are giving the AP Physics 1: Algebra-Based Exam, say:


Section II has 5 questions. It is suggested that you spend approximately
25 minutes each for questions 2 and 3, and 13 minutes each for
questions 1, 4, and 5.

If you are giving the AP Physics 2: Algebra-Based Exam, say:


Section II has 4 questions. It is suggested that you spend approximately
25 minutes each for questions 2 and 3, and 20 minutes each for
questions 1 and 4.

The orange booklet has reference material only. You may make notes in
the orange booklet, but no credit will be given for any work written in the
orange booklet.

210 AP Physics 1 and Physics 2 Exams


© 2021 College Board. Visit College Board on the web: collegeboard.org.
2020-21 AP Exam Instructions

You must write your responses in the free-response booklet. You must use a

AP Physics 1 and Physics 2 Exams


pen with black or dark blue ink or a No. 2 pencil. If you use a pencil, be sure that
your writing is dark enough to be easily read.
The questions are printed in the free-response booklet. The question number at
the top of the page indicates which question to answer on that page. Write your
responses in the space provided for each question.
If you run out of space, raise your hand.
Once final time is called, stop working immediately.
Are there any questions? . . .
Open both booklets and begin.
Note Start Time . Note Stop Time .

Check that students are writing their answers in their free-response booklet, not in their
orange booklet.

Make sure that students are using their calculators appropriately. You should also make
sure that Hewlett-Packard calculators’ infrared ports are not facing each other and that
students are not sharing calculators.

If a student runs out of space and raises their hand, give them extra paper and instruct
them to write the following on the top of each sheet they use:

ƒƒ their AP ID,
ƒ the exam title, and
ƒ the question number they are working on.
They must not write their name.

After 1 hour and 20 minutes, say:


There are 10 minutes remaining.

After 10 minutes, say:


Stop working and close both booklets. Place them faceup on your desk. Keep
your booklets separate; don’t put one inside the other. . . .
If any students used extra paper for a question in the free-response section, have those
students staple the extra sheet(s) to the first page corresponding to that question in their
free-response booklets. Complete an Incident Report after the exam and return these
free-response booklets with the extra sheets attached in the Incident Report return envelope
(see page 270).

Then say:
Remain in your seat, without talking, while the exam materials are collected.
Collect a free-response booklet and an orange booklet from each student. Check for
the following:

ƒƒ Free-response booklet front cover: The student printed their initials and today’s date and
placed an AP ID label on the shaded box at the bottom.
ƒ Free-response booklet back cover: The student completed the “Important Identification
Information” area.
ƒ The student wrote responses in the free-response booklet and not in the orange booklet.
If any students wrote their responses in the orange booklet instead of the free-response
booklet, refer to the Administration Incidents table for instructions. (See “Answers written
in an orange booklet instead of in the free-response booklet” on page 269.)

AP Physics 1 and Physics 2 Exams  211


© 2021 College Board. Visit College Board on the web: collegeboard.org.
2020-21 AP Exam Instructions

The orange booklet must be returned with the rest of your exam materials. Keep the orange
booklets separate from the free-response booklets. Do not place free-response booklets
inside the orange booklets or vice versa.

Then say:
I have a few last important reminders:
You can have one AP score report sent for free. June 20 is the deadline to
indicate or change the college, university, or scholarship program that you’d
like to receive your free score report. This needs to be done through “My AP.”

If you are giving the Administration 1 exam, say:


You may not discuss or share the free-response exam content with anyone
unless it is released on the College Board website in about two days. Your AP
Exam scores for this year will be available online in July.

If you are giving the Administration 2 exam say:


None of the content in this exam may ever be discussed or shared in any way at
any time. Your AP Exam scores for this year will be available online in July.

When all exam materials have been collected and accounted for, return to students any
electronic devices you may have collected before the start of the exam.

Then say:
You are now dismissed.

After-Exam Tasks
Return the AP ID label sheets to the AP coordinator so the label sheets can be organized
for each upcoming exam administration. Keep in mind that the schedule printed on the
label sheet may not reflect recent changes to a student’s exam schedule. If you need to
confirm a student’s exam schedule, reference the AP coordinator’s master exam list.

Be sure to give the completed seating chart to the AP coordinator. Schools must retain
seating charts for at least six months (unless the state or district requires that they be
retained for a longer period of time). Schools should not return any seating charts in their
exam shipments unless they are required as part of an Incident Report.

NOTE: If you administered exams to students with accommodations, review Part 2 of the
2020-21 AP Coordinator’s Manual and the 2020-21 AP SSD Guidelines for information about
completing the Nonstandard Administration Report (NAR) form and returning these exams.

The exam proctor should complete the following tasks if asked to do so by the
AP coordinator. Otherwise, the AP coordinator must complete these tasks:

ƒƒ Complete an Incident Report for any students who used extra paper for the
free-response section. (Incident Report forms are provided in the coordinator packets
sent with the exam shipments.) These forms must be completed with a No. 2 pencil.
It is best to complete a single Incident Report for multiple students per exam subject,
per administration, as long as all required information is provided. Include all exam
booklets with extra sheets of paper in an Incident Report return envelope (see page 270).
ƒ Return all exam materials to secure storage until they are shipped back to the
AP Program. (See “Storing Exam Materials” in Part 2 of the 2020-21 AP Coordinator’s
Manual for more information about secure storage.)

212 AP Physics 1 and Physics 2 Exams


© 2021 College Board. Visit College Board on the web: collegeboard.org.
Name:____________________________________

Answer Sheet for AP Physics 1: Algebra-Based


Practice Exam, Section I

No. Answer No. Answer


1 26
2 27
3 28
4 29
5 30
6 31
7 32
8 33
9 34
10 35
11 36
12 37
13 38
14 39
15 40
16 41
17 42
18 43
19 44
20 45
21 131
22 132
23 133
24 134
25 135
®
AP Physics 1: Algebra-Based Exam
SECTION I: Multiple Choice

DO NOT OPEN THIS BOOKLET UNTIL YOU ARE TOLD TO DO SO.

Instructions
At a Glance
Section I of this exam contains 50 multiple-choice questions. Pages containing equations
Total Time and other information are also printed in this booklet. Calculators, rulers, and
1 hour and 30 minutes straightedges may be used in this section.
Number of Questions
50 Indicate all of your answers to the multiple-choice questions on the answer sheet. No
Percent of Total Score credit will be given for anything written in this exam booklet, but you may use the booklet
50% for notes or scratch work.
Writing Instrument
Pencil required Because this section offers only four answer options for each question, do not mark the
Electronic Device (E) answer circle for any question. If you change an answer, be sure that the previous
Calculator allowed mark is erased completely.
For questions 1 through 45, select the single best answer choice for each question. After
you have decided which of the choices is best, completely fill in the corresponding circle
on the answer sheet. Here is a sample question and answer.

For questions 131 through 135, select the two best answer choices for each question.
After you have decided which two choices are best, completely fill in the two
corresponding circles on the answer sheet. Here is a sample question and answer.

Use your time effectively, working as quickly as you can without losing accuracy. Do not
spend too much time on any one question. Go on to other questions and come back to
the ones you have not answered if you have time. It is not expected that everyone will
know the answers to all of the multiple-choice questions.
Your total score on Section I is based only on the number of questions answered correctly.
Points are not deducted for incorrect answers or unanswered questions.
AP® PHYSICS 1 TABLE OF INFORMATION
CONSTANTS AND CONVERSION FACTORS
Proton mass, mp  1.67  1027 kg Electron charge magnitude, e  1.60  10 19 C
Neutron mass, mn  1.67  10 27 kg Coulomb’s law constant, k  1 4 pe0  9.0  10 9 N  m 2 C2
Universal gravitational
Electron mass, me  9.11  10 31 kg 11
m 3 kgs2
constant, G  6.67  10
Acceleration due to gravity
Speed of light, c  3.00  108 m s at Earth’s surface, g  9.8 m s
2

meter, m kelvin, K watt, W degree Celsius, C


UNIT kilogram, kg hertz, Hz coulomb, C
SYMBOLS second, s newton, N volt, V
ampere, A joule, J ohm, W

PREFIXES VALUES OF TRIGONOMETRIC FUNCTIONS FOR COMMON ANGLES


Factor Prefix Symbol       
q 0 30 37 45 53 60 90
12 tera T
10
sinq 0 12 35 2 2 45 3 2 1
10 9 giga G
10 6 mega M cosq 1 3 2 45 2 2 35 12 0
10 3 kilo k tanq 0 3 3 34 1 43 3 
2 centi c
10
3 milli m
10 The following conventions are used in this exam.
10 6 micro m I. The frame of reference of any problem is assumed to be inertial unless
otherwise stated.
9 nano n
10 II. Assume air resistance is negligible unless otherwise stated.
10 12 pico p III. In all situations, positive work is defined as work done on a system.
IV. The direction of current is conventional current: the direction in which
positive charge would drift.
V. Assume all batteries and meters are ideal unless otherwise stated.
AP® PHYSICS 1 EQUATIONS
MECHANICS ELECTRICITY
à x  Ãx 0  a x t a = acceleration  qq A = area
A = amplitude FE  k 1 2 2 F = force
r
1 2 d = distance I = current
x  x0  Ãx 0 t  at Dq
2 x E = energy I   = length
Dt
f = frequency P = power
Ãx2  Ãx20  2 a x  x  x0  F = force R
r
q = charge
  I = rotational inertia A R = resistance
 F Fnet = kinetic energy DV = separation
a  K I  r
m m k = spring constant R t = time
  L = angular momentum P  I DV V = electric potential
Ff  m Fn
 = length Rs   Ri r = resistivity
2 m = mass i
à 1 1
ac 
r
P
p
=
=
power
momentum Rp
  Ri
  i
p  mv r = radius or separation
  T = period
Dp  F Dt t = time WAVES
U = potential energy f = frequency
1 v
K  mv2 V = volume l  v = speed
2 v = speed
f
l = wavelength
DE  W  Fd  Fd cos q W = work done on a system
GEOMETRY AND TRIGONOMETRY
x = position
DE y = height Rectangle A = area
P a = angular acceleration A  bh C = circumference
Dt
m = coefficient of friction V = volume
1 2 Triangle S = surface area
q  q0  w0t  at q = angle
2 1 b = base
r = density A bh
2 h = height
w  w0  at t = torque  = length
w = angular speed Circle w = width
x  A cos 2 p ft 
A  pr 2 r = radius
  DUg  mg Dy
 t t net C  2 pr
a 
I I
2p 1 Rectangular solid Right triangle
T  
t  r F  rF sin q w f V  wh c 2  a 2  b2
L  Iw a
Ts  2 p
m Cylinder sin q 
k c
DL  t Dt V  pr 2
b
 S  2 pr   2 pr 2 cos q 
1 Tp  2 p c
K  Iw2 g a
2 Sphere tan q 
  mm b
 Fg  G 1 2 2 4 3
Fs  k x V  pr
r 3 c a
 90°
Us 
1 2
kx  Fg S  4pr 2 q
2 g  b
m
m
r UG  
Gm1m2
V r
PHYSICS 1
SECTION I
Time—1 hour and 30 minutes
50 Questions

Note: To simplify calculations, you may use g  10 m/s2 in all problems.

Directions: Each of the questions or incomplete statements below is followed by four suggested answers or
completions. Select the one that is best in each case and then fill in the corresponding circle on the answer sheet.

1. The figure above shows a truck pulling three crates across a rough road. Which of the following shows the
directions of all the horizontal forces acting on crate 2 ?

(A)

(B)

(C)

(D)

-4- GO ON TO THE NEXT PAGE.


2. Two blocks are on a horizontal, frictionless surface. Block A is moving with an initial velocity of v0 toward
block B, which is stationary, as shown above. The two blocks collide, stick together, and move off with a
velocity of v0 / 3. Which block, if either, has the greater mass?

(A) Block A
(B) Block B
(C) Neither; their masses are the same.
(D) The answer cannot be determined without knowing the mass of one of the blocks.

-- GO ON TO THE NEXT PAGE.


Questions 3 through 5 refer to the following material.

A student sets an object attached to a spring into oscillatory motion and uses a motion detector to record the velocity
of the object as a function of time. A portion of the recorded data is shown in the figure above.

3. The total change in the object’s speed between 5. The frequency of oscillation is most nearly
1.0 s and 1.1 s is most nearly
(A) 0.63 Hz
(A) zero (B) 0.80 Hz
(B) 5 cm / s (C) 1.25 Hz
(C) 10 cm / s (D) 1.60 Hz
(D) 15 cm / s

4. The acceleration of the object at time t = 0.7 s is


most nearly equal to which of the following?

(A) The value of the graph where it crosses the


0.7 s grid line
(B) The slope of the line connecting the origin
and the point where the graph crosses the
0.7 s grid line
(C) The area under the curve between where
the graph crosses the time axis near
0.63 s and time 0.7 s
(D) The slope of the tangent to a best-fit
sinusoidal curve at 0.7 s

-- GO ON TO THE NEXT PAGE.


Questions 6 and 7 refer to the following.

Two students need to move two identical boxes of mass M0 across a room where friction between the floor and the
boxes cannot be neglected. One student moves the first box by pushing with a force of magnitude F0 at an angle q
from the horizontal, as shown in the figure for scenario 1. The other student moves the second box by pulling with a
force of magnitude F0 at the same angle q from the horizontal, as shown in the figure for scenario 2.

-- GO ON TO THE NEXT PAGE.


6. Which of the following graphs could describe the motion of the two boxes as they are moved across the room?

(A)

(B)

(C)

(D)

-- GO ON TO THE NEXT PAGE.


7. Which of the following is a correct expression
for the acceleration of the box in SCENARIO  ?

F0 cos q − m(M0 g − F0 sin q)


(A)
M0
F0 cos q − m(M0 g + F0 sin q)
(B)
M0
F0 cos q + m(M0 g − F0 sin q)
(C)
M0
F0 cos q + m(M0 g + F0 sin q)
(D)
M0

-- GO ON TO THE NEXT PAGE.


8. A box of mass m is initially at rest at the top of
a ramp that is at an angle q with the horizontal.
The block is at a height h and length L from the
bottom of the ramp. The block is released and
slides down the ramp. The coefficient of kinetic
friction between the block and the ramp is m .
What is the kinetic energy of the box at the
bottom of the ramp?

(A) mgh
(B) mmgL cosq
(C) mgh − mmgL cosq
(D) mgL − mmgh cosq

-- GO ON TO THE NEXT PAGE.


9. During an experiment a student records the net horizontal force exerted on an object moving in a straight line
along a horizontal frictionless track. The graph above shows the force as a function of time. Of the following,
which is the best approximation of the magnitude of the change in momentum of the object between 0 s and
4s?

(A) 20 kg · m / s
(B) 30 kg · m / s
(C) 40 kg · m / s
(D) The magnitude of the change in momentum cannot be determined without knowing the mass of the
object.

11. The figure above shows the paths of two stars of


equal mass as they orbit their common center of
mass. The positions of the stars at four different
times are labeled in the figure. At which of the
10. Four different constant forces are exerted on a positions do the stars have their greatest speed?
2.0 kg object. The figure represents the
magnitude and direction of each force. If the (A) Position 2 only
object is initially at rest, how long will it take the (B) Position 4 only
object to reach a speed of 2.0 m/s ?
(C) Positions 1 and 3 only
(A) 0.20 s (D) All of the positions, since the stars move
with constant speed
(B) 1.0 s
(C) 2.0 s
(D) 4.0 s

-- GO ON TO THE NEXT PAGE.


Questions 12 through 14 refer to the following material.

A cylinder at rest is released from the top of a ramp, as shown above. The ramp is 1.0 m high, and the cylinder rolls
down the ramp without slipping. At the bottom of the ramp, the cylinder makes a smooth transition to a small section
of a horizontal table and then travels over the edge at a height of 1.0 m above the floor, eventually landing on the
floor at a horizontal distance of 1.5 m from the table.

12. As the cylinder rolls down the ramp, how do the potential energy of the cylinder-Earth system and the kinetic
energy of the cylinder change, if at all?

(A) Potential Energy of


Kinetic Energy of Cylinder
Cylinder-Earth System
Stays the same Increases

(B) Potential Energy of


Kinetic Energy of Cylinder
Cylinder-Earth System
Stays the same Decreases

(C) Potential Energy of


Kinetic Energy of Cylinder
Cylinder-Earth System
Decreases Increases

(D) Potential Energy of


Kinetic Energy of Cylinder
Cylinder-Earth System
Decreases Decreases

-- GO ON TO THE NEXT PAGE.


13. After the cylinder leaves the table, but before it lands, how do the rotational kinetic energy and translational
kinetic energy of the cylinder change, if at all?

(A)
Translational Kinetic
Rotational Kinetic Energy
Energy
Increases Increases

(B) Translational Kinetic


Rotational Kinetic Energy
Energy
Increases Stays the same

(C) Translational Kinetic


Rotational Kinetic Energy
Energy
Stays the same Increases

(D) Translational Kinetic


Rotational Kinetic Energy
Energy
Stays the same Stays the same

14. A sphere with the same mass and radius as the


original cylinder, but a smaller rotational inertia,
is released from rest from the top of the ramp. Ks
and Kc are the sphere’s and the cylinder’s total
kinetic energy at the bottom of the ramp,
respectively. How do Ks and Kc compare, and
why?

(A) Ks < Kc, because the sphere will gain less


rotational kinetic energy.
(B) Ks < Kc, because the sphere has a greater
acceleration and therefore has less time to
gain kinetic energy.
(C) Ks = Kc, because both objects accelerate at
the same rate.
(D) Ks = Kc, because the gravitational force
does equal work on each object as it rolls
down the ramp.

-- GO ON TO THE NEXT PAGE.


15. A student swings a ball on a light rod at a constant speed in a vertical circle, as shown in the figure. Which of
the following correctly ranks the magnitudes of the forces exerted by the rod on the ball F1, F2, F3, and F4
when the ball is at locations 1, 2, 3, and 4, respectively?

(A) F1 = F2 = F3 = F4
(B) (F2 = F3) > F4 > F1
(C) F4 > F1 > (F2 = F3)
(D) F1 > F4 > (F2 = F3)

17. A 0.5 kg pendulum bob is raised to 1.0 m above


16. A small block slides without friction along a the floor, as shown in the figure above. The bob
track toward a circular loop. The block has more is then released from rest. When the bob is
than enough speed to remain firmly in contact 0.8 m above the floor, its speed is most nearly
with the track as it goes around the loop. The
magnitude of the block’s acceleration at the top (A) 5 m / s
of the loop is
(B) 4 m / s
(A) zero (C) 2 m / s
(B) greater than zero but less than g (D) 1 m / s
(C) equal to g
(D) greater than g

-- GO ON TO THE NEXT PAGE.


18. A block is released from rest and slides down a frictionless ramp inclined at 30° from the horizontal. When the
block reaches the bottom, the block-Earth system has mechanical energy E 0 . The experiment is repeated, but
now horizontal and vertical forces of magnitude F are exerted on the block while it slides, as shown above.
When the block reaches the bottom, the mechanical energy of the block-Earth system

(A) is greater than E 0


(B) is equal to E 0
(C) is less than E 0
(D) cannot be determined without knowing F

19. An apple is released from rest 500 m above the


ground. Due to the combined forces of air
resistance and gravity, it has a speed of 40 m / s
when it reaches the ground. What percentage of
the initial mechanical energy of the apple-Earth
system was dissipated due to air resistance? Take
the potential energy of the apple-Earth system to
be zero when the apple reaches the ground.

(A) 16%
(B) 40%
(C) 60%
(D) 84%

-- GO ON TO THE NEXT PAGE.


Questions 20 and 21 refer to the following material.

Three identical rocks are launched with identical speeds from the top of a platform of height h 0 . The rocks are
launched in the directions indicated above.

20. Which of the following correctly relates the


magnitude vy of the vertical component of the
velocity of each rock immediately before it hits
the ground?

(A) (vy1 = vy2) > vy3


(B) vy1 > vy3 > vy2
(C) vy2 > vy3 > vy1
(D) vy1 = vy2 = vy3

21. Rock 1, of mass m, reaches a maximum height


h max after being launched. During the time
between the instant rock 1 is launched from
height and the instant it returns to height h 0 , the
work done on the rock by the gravitational force
is

(A) 0
(B) −mgh 0
(C) −mg(h max − h 0)
(D) 2mg(h max − h 0)

-- GO ON TO THE NEXT PAGE.


22. A 2.5 g marshmallow is placed in one end of a 40 cm pipe, as shown in the figure above. A person blows into
the left end of the pipe to eject the marshmallow from the right end. The average net force exerted on the
marshmallow while it is in the pipe is 0.7 N . The speed of the marshmallow as it leaves the pipe is most nearly

(A) 4.7 m / s
(B) 11 m / s
(C) 15 m / s
(D) 280 m / s

23. A meterstick is held as shown above and then


released from rest. The tabletop has negligible
friction. Which figure below best indicates the
path of the center of mass of the meterstick as it
falls?

(A)

(B)

(C)

(D)

-- GO ON TO THE NEXT PAGE.


24. Blocks X and Y are glued together and released from rest on a ramp with negligible friction, as shown in trial 1.
The blocks are then separated and connected by a light spring, as shown in trial 2. The spring is compressed
and the blocks are again released from rest on the ramp. Immediately after the blocks are released, is the net
force on the two-block system the same or different between trial 1 and trial 2 ? Immediately after the blocks
are released, is the net force on block Y the same or different between trial 1 and trial 2 ?

(A) Force on System Force on Block Y


Different Different

(B) Force on System Force on Block Y


Different The same

(C) Force on System Force on Block Y


The same Different

(D) Force on System Force on Block Y


The same The same

-- GO ON TO THE NEXT PAGE.


Interval Mechanical Energy Dissipated due to Friction
1 0

2 W2

3 W3

4 W4

25. A block slides down an inclined plane whose roughness varies. Only during time interval 1 does the block slide
with negligible friction. The coefficient of kinetic friction during time interval 3 is twice the coefficient of
kinetic friction during time interval 2. The graph shows the block’s velocity as a function of time. The table
lists the mechanical energy dissipated due to friction over the different time intervals. Which of the following
claims is true?

(A) W4 > W3 > W2


(B) W2 = W3 = W4
(C) W3 > W2 > W4

(D) ( W2 = W4 )> W3

-- GO ON TO THE NEXT PAGE.


26. Two identical objects, X and Y, move toward each other at different speeds on a horizontal surface with
negligible friction, as shown in the top figure. The objects then collide elastically and move away from each
other. The kinetic energy of object X as a function of time is shown in the graph. Which of the following is true
of the speed vY of object Y?

(A) vY after the collision is greater than it was before the collision.
(B) vY after the collision is equal to what it was before the collision.
(C) vY after the collision is less than it was before the collision.
(D) vY after the collision cannot be compared to what it was before the collision without knowing the mass of
the objects.

27. The figure above represents a stick of uniform density that is attached to a pivot at the right end and has equally
spaced marks along its length. Any one of the four forces shown can be exerted on the stick as indicated. Which
force will create the largest rate of change in the stick’s angular momentum?

(A) The 30 N force


(B) The 40 N force
(C) The 60 N force
(D) The 150 N force

-- GO ON TO THE NEXT PAGE.


28. A disk with radius of 0.5 m is free to rotate
around its center without friction. A string
wrapped around the disk is pulled, as shown
above, exerting a 2 N force tangent to the edge
of the disk for 1 s. If the disk starts from rest,
what is its angular speed after 1 s ?

(A) 0 rad / s
(B) 1 rad / s
(C) 4 rad / s
(D) It cannot be determined without knowing
the rotational inertia of the disk.

29. The figure above shows a rod that is fixed to a horizontal surface at pivot P. The rod is initially rotating without
friction in the counterclockwise direction. At time t , three forces of equal magnitude are applied to the rod as
shown. Which of the following is true about the angular speed and direction of rotation of the rod immediately
after time t ?

(A) Angular Speed Direction of Rotation


Decreasing Counterclockwise

(B) Angular Speed Direction of Rotation


Decreasing Clockwise

(C) Angular Speed Direction of Rotation


Increasing Counterclockwise

(D) Angular Speed Direction of Rotation


Increasing Clockwise

-- GO ON TO THE NEXT PAGE.


30. Two systems are in oscillation: a simple
pendulum swinging back and forth through a
very small angle and a block oscillating on a
spring. The block-spring system takes twice as
much time as the pendulum to complete one
oscillation. Which of the following changes
could make the two systems oscillate with the
same period?

(A) Increasing the mass of the pendulum bob


(B) Increasing the angle through which the
pendulum swings by a small amount
(C) Decreasing the mass of both the block and
the pendulum bob
(D) Shortening the pendulum

-- GO ON TO THE NEXT PAGE.


Questions 31 and 32 refer to the following.

The position as a function of time for two objects moving along a straight line is shown in the graph.

31. At which of the following times do the two


objects have the same velocity?

(A) tA
(B) tB
(C) tC
(D) tD

32. Which statement is true about the distances the


two object have traveled at time t f ?

(A) Object 1 has traveled a greater distance.


(B) Object 2 has traveled a greater distance.
(C) Both objects have traveled the same
distance.
(D) The total distance traveled by each object
cannot be compared using the graph.

-- GO ON TO THE NEXT PAGE.


33. A hollow plastic ball is projected into the air.
There is significant air resistance opposing the
ball’s motion, so the magnitude of the ball’s
acceleration is not equal to g . At time t , the ball
is moving up and to the right at an angle of 45°
to the horizontal, as shown above. Which of the
following best shows the magnitude a and the
direction of the ball’s acceleration at time t ?

(A)

(B)

(C)

(D)

-- GO ON TO THE NEXT PAGE.


34. At time t = 0 two figure skaters are moving together over ice with negligible friction, as shown above. Skater
1, represented by the large black dot, is twice as massive as skater 2 , represented by the gray dot. At t = 2 s the
skaters push off of one another. The location of skater 1 is shown at t = 4 s. At t = 4 s, skater 2 is located at
which of the labeled points?

(A) Point A
(B) Point B
(C) Point C
(D) Point D

35. A regular hexagon of uniform density is initially


at rest and free to rotate about an axis at its
center. Three forces, FA, FB , and FC , have equal
magnitude and are able to be applied in the
direction and at the point shown in the figure.
Which of these forces, if applied individually
and for the same time interval, will produce the
greatest increase in the hexagon’s angular
speed?

(A) Force FA
(B) Force FB
(C) Force FC
(D) All three forces produce the same change
in the hexagon’s angular speed.

-- GO ON TO THE NEXT PAGE.


36. The graphs above show the magnitude F of a force exerted on an object as a function of the object’s position x
for two trials in an experiment. W1 and W2 are the work done on the object by force 1 and force 2, respectively.
How do W1 and W2 compare, and why?

(A) W1 > W2, because the maximum value of force 1 is greater than the maximum value of force 2.
(B) W1 > W2, because the slope of force 1’s graph increases, while the slope of force 2’s graph decreases.
(C) W1 < W2, because the average value of force 1 is smaller than the average value of force 2.
(D) W1 < W2, because at the midpoint, x = 0 . 5 m , the value of force 1 is less than the value of force 2.

37. Two identical cars, car 1 and car 2, are moving in opposite directions on a straight road. The position of each
car as a function of time is represented in the graph. What is the speed of the center of mass of the two-car
system?

(A) Zero
(B) 10 m/s
(C) 20 m/s
(D) 40 m/s

-- GO ON TO THE NEXT PAGE.


38. Two identical spheres are attached to opposite
ends of a thin rod of negligible mass, as shown
in the figure. The spheres-rod system is thrown
straight upward and is rotating counterclockwise
as it moves through the air. Which of the
following is correct about the angular
momentum of the spheres-rod system while it is
in the air? What is a valid justification?

(A) The angular momentum remains constant


because there is never a net torque acting
on the system.
(B) The angular momentum remains constant
because there is a changing net torque
acting on the system that averages to zero
over time.
(C) The angular momentum constantly
decreases because there is a constant net
torque acting on the system.
(D) The angular momentum constantly
increases because there is a constant net
torque acting on the system.

-- GO ON TO THE NEXT PAGE.


39. A 2 kg box is pushed from rest across a horizontal surface where friction between the box and the surface is
negligible. Students record the applied force at several equally spaced positions while pushing the box and
create the graph shown. The change in momentum of the box is most nearly

(A) 2 kg · m / s
(B) 6 kg · m / s
(C) 12 kg · m / s
(D) 36 kg · m / s

-- GO ON TO THE NEXT PAGE.


40. Block 1 slides rightward on the floor toward an ideal spring attached to block 2 , as shown. At time t1,
block 1 reaches the spring and starts compressing it as block 2 also starts to slide to the right. At a later time,
t2, block 1 loses contact with the spring. Both blocks slide with negligible friction. Taking rightward as
positive, which pair of graphs could represent the acceleration of block 2 and the center-of-mass acceleration
of the two-block system?

(A)

(B)

(C)

(D)

-- GO ON TO THE NEXT PAGE.


41. On a straight horizontal track along which blocks can slide with negligible friction, block 1 slides toward block
2, which is initially at rest. Block 1 collides with an electronic force probe attached to block 2, generating a
force vs. time graph and causing block 2 to start sliding. What additional measurements must be made to
determine the momentum of block 2 after the collision?

(A) None
(B) The mass of block 2 only
(C) The post-collision speed of block 2 only
(D) Both the mass and the post-collision speed of block 1

42. Three identical forces of magnitude F0 are applied to a meterstick that rests on a horizontal table, as shown in
the diagram. At what location on the meterstick would a fourth force, also of magnitude F0, need to be applied
in order to establish static equilibrium?

(A) A
(B) B
(C) C
(D) D

-
-- GO ON TO THE NEXT PAGE.
43. Two carts of masses 1 kg and 2 kg travel together, as shown in Figure 1. At time t = 2 s, a compressed spring
inside the 2 kg cart expands, and the carts separate. Figure 2 shows a graph of the carts’ velocities as a
function of time. What is the magnitude of the change in linear momentum, if any, of the two-cart system
between t = 0 s and t = 4 s ?

(A) 0 kg · m / s
(B) 0.5 kg · m / s
(C) 1.0 kg · m / s
(D) 2.0 kg · m / s

-- GO ON TO THE NEXT PAGE.


44. A cart of mass m rolls past the circular bottom of a hill (point P). Which of the following statements about the
normal force Fn exerted on the cart at point P is correct?

(A) Fn = mg, because the normal force on an incline is equal to mg cosq and q = 0 at point P.
(B) Fn = mg, because the speed of the cart is neither increasing nor decreasing at point P.
(C) Fn is equal to the centripetal force on the cart at point P, because only the track can exert a force toward
the center of the circle.
(D) Fn is greater than mg at point P, because the cart is experiencing an upward acceleration.

45. A meterstick with a uniformly distributed mass of 0.5 kg is supported by a pivot placed at the 0.25 m mark
from the left, as shown. At the left end, a small object of mass 1.0 kg is placed at the zero mark, and a second
small object of mass 0.5 kg is placed at the 0.5 m mark. The meterstick is supported so that it remains
horizontal, and then it is released from rest. One second after it is released, what is the change in the angular
momentum of the meterstick?

(A) 0
(B) 500 kg·m2/s
(C) 1000 kg·m2/s
(D) The change in angular momentum of the meterstick cannot be determined from this information.

-- GO ON TO THE NEXT PAGE.


'LUHFWLRQV)RUHDFKRIWKHTXHVWLRQVRULQFRPSOHWHVWDWHPHQWVEHORZWZRRIWKHVXJJHVWHGDQVZHUVZLOOEHFRUUHFW
)RUHDFKRIWKHVHTXHVWLRQV\RXPXVWVHOHFWERWKFRUUHFWFKRLFHVWRHDUQFUHGLW 1RSDUWLDOFUHGLWZLOOEHHDUQHGLI
RQO\RQHFRUUHFWFKRLFHLVVHOHFWHG 6HOHFWWKHWZRWKDWDUHEHVWLQHDFKFDVHDQGWKHQILOOLQWKHFRUUHVSRQGLQJFLUFOHV
WKDWEHJLQZLWKQXPEHURQWKHDQVZHUVKHHW

 A water-skier with weight Fg = mg moves to the


right with acceleration a . A horizontal tension
force T is exerted on the skier by the rope, and a
horizontal drag force Fd is exerted by the water
on the ski. The water also exerts a vertical lift
force L on the skier. Which of the following are
correct relationships between the forces exerted
on the skier-ski system? Select two answers.

(A) T − Fd = ma
(B) L − Fg = ma
(C) L − Fg = 0
(D) T − Fd = 0

-- GO ON TO THE NEXT PAGE.


 Two model cars, A and B, have the same mass but different bumpers. The acceleration of each car during its
collision with a wall is measured, and the data are shown in the graphs above. Which of the following
statements about the collisions are correct? Select two answers.

(A) Both cars reach their maximum speed at 10.05 s.


(B) The cars experience approximately the same impulse.
(C) Car B experiences a nonzero force for a longer time than car A.
(D) The change in momentum for car B occurs over a shorter period of time than for car A.

-- GO ON TO THE NEXT PAGE.


133. The graph above shows the speed of a truck as it moves along a straight, level road. Which of the following
describe a method to determine the distance d the truck travels during the 10 s time interval shown? Select two
answers.

(A) Multiply the average speed of 18 m / s by the 10 s travel time.


(B) Multiply half the initial speed of 30 m / s by the 10 s travel time.
(C) Calculate the slope of the line in the graph to determine the acceleration a and then use d = (1 / 2)at 2 ,
where t is the travel time.
(D) Calculate the area under the line in the graph.

134. A vehicle lands on Mars and explores its


surface. The average gravitational field on the
surface of Mars is 3.7 N / kg. The weight of the
vehicle is defined as the gravitational force
exerted on it. Which of the following statements
are true about the vehicle’s weight? Select two
answers.

(A) The vehicle’s weight was constant until it


reached the surface of Mars.
(B) The vehicle’s weight increased while it was
descending to the surface of Mars.
(C) The vehicle’s weight always equals the
normal force exerted by Mars on the
vehicle while it is landing.
(D) The vehicle weighs less on the surface of
Mars than on the surface of Earth.

-- GO ON TO THE NEXT PAGE.


 The energy bar charts shown in the figure indicate the gravitational potential energy U and total kinetic energy
K for a system at two different times, t1 and a later time t2. The tables describe four different situations.

For which of the situations could the energy bar charts accurately depict U and K at the two times? Select two
answers.

(A) Situation A
(B) Situation B
(C) Situation C
(D) Situation D

-- GO ON TO THE NEXT PAGE.


END OF SECTION I

IF YOU FINISH BEFORE TIME IS CALLED, YOU MAY


CHECK YOUR WORK ON THIS SECTION.

DO NOT GO ON TO SECTION II UNTIL YOU ARE TOLD TO DO SO.


____________________________________________________________________________

MAKE SURE YOU HAVE DONE THE FOLLOWING:

• PLACED YOUR AP ID LABEL ON YOUR ANSWER SHEET


• WRITTEN AND GRIDDED YOUR AP ID CORRECTLY ON YOUR
ANSWER SHEET
• TAKEN THE AP EXAM LABEL FROM THE FRONT OF THIS BOOKLET AND
PLACED IT ON YOUR ANSWER SHEET

-3-
®
AP Physics 1: Algebra-Based Exam
SECTION II: Free Response
DO NOT OPEN THIS BOOKLET UNTIL YOU ARE TOLD TO DO SO. on Information
PLEASE PRINT WITH PEN:
1. First two letters of your last name 4. Unless I fill in the circle below, I grant
At a Glance College Board the unlimited right to use,
First letter of your first name reproduce, and publish my free-response
Total Time materials, both written and oral, for educational
1 hour and 30 minutes research and instructional purposes. My name
Number of Questions 2. Date of birth and the name of my school will not be used in
5 any way in connection with my free-response
Percent of Total Score Month Day
materials. I understand that I am free to
50% mark “No” with no effect on my score or
3. Six-digit school code its reporting.
Writing Instrument
Either pencil or pen with No, I do not grant College Board
black or dark blue ink Month Day these rights.
Electronic Device
Calculator allowed
Suggested Time
Approximately
25 minutes each for
questions 2 and 3 and
13 minutes each for Instructions
questions 1, 4, and 5
Weight The questions for Section II are printed in this booklet. A table of information and lists of
Approximate weights: equations that may be helpful are printed in the orange Reference booklet. You may use
Questions 2 and 3: any blank space in the orange booklet for scratch work, but you must write your answers
26% each in the spaces provided for each answer in this Free Response booklet. No credit will be
Questions 1, 4, and 5:
16% each given for any work written in the orange booklet. Calculators, rulers, and straightedges
may be used in this section.
All final numerical answers should include appropriate units. Credit for your work
depends on demonstrating that you know which physical principles would be appropriate
to apply in a particular situation. Therefore, you should show your work for each part in
the space provided after that part. If you need more space, be sure to clearly indicate
where you continue your work. Credit will be awarded only for work that is clearly
designated as the solution to a specific part of a question. Credit also depends on the
quality of your solutions and explanations, so you should show your work.
Write clearly and legibly. Do not write outside the box. Cross out any errors you make;
erased or crossed-out work will not be scored. You may lose credit for incorrect work that
is not crossed out.
Manage your time carefully. You may proceed freely from one question to the next. You
may review your responses if you finish before the end of the exam is announced.
AP® PHYSICS 1 TABLE OF INFORMATION
CONSTANTS AND CONVERSION FACTORS
Proton mass, mp  1.67  1027 kg Electron charge magnitude, e  1.60  10 19 C
Neutron mass, mn  1.67  10 27 kg Coulomb’s law constant, k  1 4 pe0  9.0  10 9 N  m 2 C2
Universal gravitational
Electron mass, me  9.11  10 31 kg 11
m 3 kgs2
constant, G  6.67  10
Acceleration due to gravity
Speed of light, c  3.00  108 m s at Earth’s surface, g  9.8 m s
2

meter, m kelvin, K watt, W degree Celsius, C


UNIT kilogram, kg hertz, Hz coulomb, C
SYMBOLS second, s newton, N volt, V
ampere, A joule, J ohm, W

PREFIXES VALUES OF TRIGONOMETRIC FUNCTIONS FOR COMMON ANGLES


Factor Prefix Symbol       
q 0 30 37 45 53 60 90
12 tera T
10
sinq 0 12 35 2 2 45 3 2 1
10 9 giga G
10 6 mega M cosq 1 3 2 45 2 2 35 12 0
10 3 kilo k tanq 0 3 3 34 1 43 3 
2 centi c
10
3 milli m
10 The following conventions are used in this exam.
10 6 micro m I. The frame of reference of any problem is assumed to be inertial unless
otherwise stated.
9 nano n
10 II. Assume air resistance is negligible unless otherwise stated.
10 12 pico p III. In all situations, positive work is defined as work done on a system.
IV. The direction of current is conventional current: the direction in which
positive charge would drift.
V. Assume all batteries and meters are ideal unless otherwise stated.
AP® PHYSICS 1 EQUATIONS
MECHANICS ELECTRICITY
à x  Ãx 0  a x t a = acceleration  qq A = area
A = amplitude FE  k 1 2 2 F = force
r
1 2 d = distance I = current
x  x0  Ãx 0 t  at Dq
2 x E = energy I   = length
Dt
f = frequency P = power
Ãx2  Ãx20  2 a x  x  x0  F = force R
r
q = charge
  I = rotational inertia A R = resistance
 F Fnet = kinetic energy DV = separation
a  K I  r
m m k = spring constant R t = time
  L = angular momentum P  I DV V = electric potential
Ff  m Fn
 = length Rs   Ri r = resistivity
2 m = mass i
à 1 1
ac 
r
P
p
=
=
power
momentum Rp
  Ri
  i
p  mv r = radius or separation
  T = period
Dp  F Dt t = time WAVES
U = potential energy f = frequency
1 v
K  mv2 V = volume l  v = speed
2 v = speed
f
l = wavelength
DE  W  Fd  Fd cos q W = work done on a system
GEOMETRY AND TRIGONOMETRY
x = position
DE y = height Rectangle A = area
P a = angular acceleration A  bh C = circumference
Dt
m = coefficient of friction V = volume
1 2 Triangle S = surface area
q  q0  w0t  at q = angle
2 1 b = base
r = density A bh
2 h = height
w  w0  at t = torque  = length
w = angular speed Circle w = width
x  A cos 2 p ft 
A  pr 2 r = radius
  DUg  mg Dy
 t t net C  2 pr
a 
I I
2p 1 Rectangular solid Right triangle
T  
t  r F  rF sin q w f V  wh c 2  a 2  b2
L  Iw a
Ts  2 p
m Cylinder sin q 
k c
DL  t Dt V  pr 2
b
 S  2 pr   2 pr 2 cos q 
1 Tp  2 p c
K  Iw2 g a
2 Sphere tan q 
  mm b
 Fg  G 1 2 2 4 3
Fs  k x V  pr
r 3 c a
 90°
Us 
1 2
kx  Fg S  4pr 2 q
2 g  b
m
m
r UG  
Gm1m2
V r
Begin your response to QUESTION 1 on this page.

1. (7 points, suggested time 13 minutes)

A toy consists of two identical solid spheres connected by a string with negligible mass. The toy is thrown at an
angle above the horizontal (not straight up) such that the string remains taut and both spheres are revolving
counterclockwise in a vertical plane around the center of the string, as shown above.

(a) Sketch graphs of the horizontal and vertical components of the velocity of the center of the string as a function
of time, from the instant the spheres are released at time t = 0 until the instant the system returns to its initial
height at time t f . Take the positive direction to be toward the right for the horizontal component and the
positive direction to be upward for the vertical component.

Unauthorized copying or reuse of this page is illegal. Page 2 GO ON TO THE NEXT PAGE.

Use a pencil or a pen with black or dark blue ink. Do NOT write your name. Do NOT write outside the box.
Continue your response to QUESTION 1 on this page.

(b) The figure above shows the toy at the instant the center of the string reaches the top of its trajectory. This is a
side view: the sphere on the left is higher than the sphere on the right.
i. On the dot below, which represents the left sphere only, draw and label the forces (not components) exerted
on the left sphere at this instant. Represent each force by a distinct arrow starting on, and pointing away from,
the dot. The dashed line is drawn at the same angle as the string.

ii. On the dot below, which represents the whole toy (the spheres-string system), draw and label the
forces (not components) that act on the system at this instant. Represent each force by a distinct arrow starting
on, and pointing away from, the dot. The dashed line is drawn at the same angle as the string.

Unauthorized copying or reuse of this page is illegal. Page 3 GO ON TO THE NEXT PAGE.

Use a pencil or a pen with black or dark blue ink. Do NOT write your name. Do NOT write outside the box.
Continue your response to QUESTION 1 on this page.

iii. When the toy was released, the center of the string was moving with an initial speed of 15 m / s at a 60° angle
above the horizontal. Calculate the speed of the center of the string at the instant shown above, when the center
of the string reaches the top of its trajectory.

Unauthorized copying or reuse of this page is illegal. Page 4 GO ON TO THE NEXT PAGE.

Use a pencil or a pen with black or dark blue ink. Do NOT write your name. Do NOT write outside the box.
Begin your response to QUESTION 2 on this page.

2. (12 points, suggested time 25 minutes)

A heavy lab cart moves with kinetic energy K init on a track and collides with a lighter lab cart that is initially at

rest. The carts bounce off each other but the collision is not perfectly elastic, causing the two-cart system to lose

kinetic energy Klost. A student wonders if the fraction of kinetic energy lost from the two-cart system during the

⎛ Klost ⎞
collision ⎜⎜ ⎟⎟ depends on the speed of the first cart before the collision and plans to perform an experiment.
⎜⎝ K init ⎟⎠⎟

(a) The student hypothesizes that a greater fraction of kinetic energy is lost from the system during the collision
when the speed of the first cart is greater.
Briefly state one reason the hypothesis might be correct.

(b) Design an experimental procedure that could be used to test the student’s hypothesis. Assume equipment
usually found in a school physics laboratory is available.

In the table below, list the quantities that would be measured and the equipment that would be used to measure
each quantity. Also, define a symbol to represent each quantity. You do not need to use every row and may add
additional rows as needed.

Equipment for
Quantity to be Measured Symbol
Measurement

Describe the overall procedure to be used, referring to the table above. Provide enough detail so that another
student could replicate the experiment. As needed, use the symbols defined in the table and/or include a simple
diagram of the setup. Be sure to address how experimental uncertainty could be reduced.

Unauthorized copying or reuse of this page is illegal. Page 5 GO ON TO THE NEXT PAGE.

Use a pencil or a pen with black or dark blue ink. Do NOT write your name. Do NOT write outside the box.
Continue your response to QUESTION 2 on this page.

(c) Describe how the experimental data could be analyzed to confirm or disconfirm the hypothesis that a greater
fraction of kinetic energy is lost from the system during the collision when the speed of the first cart is greater.
Include a description or example of any equations, data tables, graphs, or other representations that could be
used.

(d) Consider a different scenario in which the carts stick together after the collision. The masses of the heavier and

⎛K ⎞
lighter cart are m1 and m 2 respectively. Derive an expression for the fraction of kinetic energy lost ⎜⎜ lost ⎟⎟⎟
⎜⎜⎝ K init ⎟⎠

during the collision. Express your answer in terms of m1 and m 2.

Unauthorized copying or reuse of this page is illegal. Page 6 GO ON TO THE NEXT PAGE.

Use a pencil or a pen with black or dark blue ink. Do NOT write your name. Do NOT write outside the box.
Begin your response to QUESTION 3 on this page.

3. (12 points, suggested time 25 minutes)

A crane and box are both initially at rest. At time t = 0 s, the crane begins to drive forward at a constant speed of
m m
0.5 , while also lifting the box with an upward acceleration of 1 2 . The box does not swing while being lifted
s s
by the crane.

(a) On the dot below, which represents the box, draw and label the forces (not components) that are exerted on the
box during the scenario described above. Represent each force by a distinct arrow starting on, and pointing
away from, the dot.

Unauthorized copying or reuse of this page is illegal. Page 7 GO ON TO THE NEXT PAGE.

Use a pencil or a pen with black or dark blue ink. Do NOT write your name. Do NOT write outside the box.
Continue your response to QUESTION 3 on this page.

Note: Draw the relative lengths of all vectors to reflect the relative magnitudes of all the forces.

Unauthorized copying or reuse of this page is illegal. Page 8 GO ON TO THE NEXT PAGE.

Use a pencil or a pen with black or dark blue ink. Do NOT write your name. Do NOT write outside the box.
Continue your response to QUESTION 3 on this page.

(b) On the grid below, sketch the shape of the path taken by the box as it is lifted by the crane as viewed by a
stationary observer.

Justify the shape of the path you drew above.

(c) Assume that, at t = 0 s , the horizontal position of the box is x = 0 and it has a vertical position of y = 0 .
Derive an equation that describes the vertical position y of the box as a function of the horizontal position x
of the box.

(d) Does your equation from part (c) agree with your sketch in part (b)? Justify your response.

Unauthorized copying or reuse of this page is illegal. Page 9 GO ON TO THE NEXT PAGE.

Use a pencil or a pen with black or dark blue ink. Do NOT write your name. Do NOT write outside the box.
Continue your response to QUESTION 3 on this page.

At time t = t1, the crane begins to accelerate forward, reaching a new constant velocity at time t = t2. The box
accelerates upward during the entire time interval t1 < t < t2 .

(e) The dot below represents the box during the time interval t1 < t < t2 . On the dot, draw an arrow indicating the
direction of the net force exerted on the block during this interval. The arrow should start on, and point away
from, the dot.

Unauthorized copying or reuse of this page is illegal. Page 10 GO ON TO THE NEXT PAGE.

Use a pencil or a pen with black or dark blue ink. Do NOT write your name. Do NOT write outside the box.
Begin your response to QUESTION 4 on this page.

4. (7 points, suggested time 13 minutes)

A student strikes a block at the bottom of a ramp, giving it an initial speed v0 up the ramp, as shown. There is
friction between the ramp and the block as it slides a distance x up the ramp and then slides back down.

(a) On the dots below, which represent the block as it is sliding up the ramp and down the ramp, draw and label
the forces (not components) exerted on the block. Represent each force by a distinct arrow starting on, and
pointing away from, the dot. The dashed lines are drawn at the same angle as the surface of the ramp.

(b) The block takes time t up to slide up the ramp a distance x . The block then takes time tdown to slide back down
to the bottom of the ramp, where it has speed vf . Is tdown greater than, equal to, or less than t up ?

___tdown > t up ___tdown = t up ___ tdown < t up

In a clear, coherent paragraph-length response that may also contain figures and/or equations, explain your
reasoning. If you need to draw anything other than what you have shown in part (a) to assist in your response, use
the space below. Do NOT add anything to the figures in part (a).

Unauthorized copying or reuse of this page is illegal. Page 11 GO ON TO THE NEXT PAGE.

Use a pencil or a pen with black or dark blue ink. Do NOT write your name. Do NOT write outside the box.
Begin your response to QUESTION 5 on this page.

5. (7 points, suggested time 13 minutes)

A student in a physics lab drops a ball of mass 0.50 kg from a height of 1.5 m onto a force plate. The ball hits the
force plate and rebounds upward. The student records the maximum height of the ball after it rebounds to
be 1.1 m .

(a) The student uses their observations to correctly claim that the collision between the ball and the force plate is
inelastic. Use the student’s observations to justify their claim.

(b) Calculate the impulse exerted on the ball by the force plate.

The force plate records the following data for the magnitude of the force as a function of time for the 1 ms the ball
was in contact with the force sensor.

(c) Does the force plate data provide a reliable measurement of the impulse exerted on the ball by the force
sensor? Justify your answer

Unauthorized copying or reuse of this page is illegal. Page 12 GO ON TO THE NEXT PAGE.

Use a pencil or a pen with black or dark blue ink. Do NOT write your name. Do NOT write outside the box.
DO NOT WRITE ON THIS PAGE.

STOP

END OF EXAM

IF YOU FINISH BEFORE TIME IS CALLED,


YOU MAY CHECK YOUR WORK ON THIS SECTION.
___________________________________________

THE FOLLOWING INSTRUCTIONS APPLY TO THE COVERS OF THIS SECTION II: FREE
RESPONSE BOOKLET. MAKE SURE YOU HAVE DONE THE FOLLOWING:

• COMPLETED THE IDENTIFICATION INFORMATION AS REQUESTED ON THE


FRONT AND BACK COVERS OF THIS FREE RESPONSE BOOKLET
• CHECKED THAT YOUR AP ID LABEL IS IN THE BOX ON THE FRONT COVER

Page 13
Answer Key for AP Physics 1: Algebra-Based
Practice Exam, Section I

Question 1: C Question 26: A


Question 2: B Question 27: C
Question 3: B Question 28: D
Question 4: D Question 29: A
Question 5: D Question 30: C
Question 6: C Question 31: C
Question 7: A Question 32: A
Question 8: C Question 33: B
Question 9: B Question 34: D
Question 10: C Question 35: A
Question 11: B Question 36: C
Question 12: C Question 37: A
Question 13: C Question 38: A
Question 14: D Question 39: C
Question 15: B Question 40: A
Question 16: D Question 41: A
Question 17: C Question 42: B
Question 18: A Question 43: A
Question 19: D Question 44: D
Question 20: A Question 45: A
Question 21: A Question 131: A, C
Question 22: C Question 132: B, D
Question 23: C Question 133: A, D
Question 24: C Question 134: B, D
Question 25: B Question 135: B, D
Multiple-Choice Section for Physics 1
Course Framework Alignment and Rationales

Question 1

Science Practice Learning Objective Topic

1.4 3.A.3.1 2.5


(A) Incorrect. There is a force of friction exerted on box 2 as well as a
force exerted on box 2 by box three. There are two forces that are
missing from this diagram.
(B) Incorrect. There is a force of friction exerted on box 2. There is one
force missing from this diagram.
(C) Correct. The cable pulls crate 2 forward (right), while the rope,
friction from the ground and crate 3 all exert forces backwards (left)
on crate 2.
(D) Incorrect. While there are the correct number of forces exerted on
box 2, box 3 will not exert a forward force on box 2.

Question 2

Science Practice Learning Objective Topic

2.2 5.D.2.5 5.4


(A) Incorrect. Because momentum is constant for the two-block system
from just before to just after the collision, if the mass of block A were
greater, we would expect the combined velocity of the two-block
system to be greater than 𝑣𝑣0 /2.
(B) Correct. Momentum of the two-block system is conserved because
no external forces act on the two-block system. The magnitude of the
change in momentum of block A will be equal to the magnitude of
the change in momentum of block B and the more massive block will
have a smaller change in velocity.
(C) Incorrect. Because momentum is constant for the two-block system
from just before to just after the collision, if the if the two masses
were equal, we would expect the combined velocity of the two-block
system to be equal to 𝑣𝑣0 /2.
(D) Incorrect. Because momentum is constant for the two-block system
from just before to just after the collision, if the mass of block B is
greater than the mass of block A, we would expect the combined
velocity of the two-block system to be less than 𝑣𝑣0 /2.
Question 3

Science Practice Learning Objective Topic

5.1 3.A.1.1 1.1


3.A.1.3
cm
(A) Incorrect. At 1.0s the object has a velocity of approximately -5 and
s
cm
at 1.1s, the object has a velocity of approximately -10 . The change
s
in speed is the absolute value of the difference of these two values.
cm
(B) Correct. At 1.0s the object has a velocity of approximately -5 and
s
cm
at 1.1s, the object has a velocity of approximately -10 . The change
s
in speed is the absolute value of the difference of these two values.
cm
(C) Incorrect. At 1.0s the object has a velocity of approximately -5 and
s
cm
at 1.1s, the object has a velocity of approximately -10 . The change
s
in speed is the absolute value of the difference of these two values.
cm
(D) Incorrect. At 1.0s the object has a velocity of approximately -5 and
s
cm
at 1.1s, the object has a velocity of approximately -10 . The change
s
in speed is the absolute value of the difference of these two values.

Question 4

Science Practice Learning Objective Topic

5.1 3.A.1.1 1.1


(A) Incorrect. The value of the graph where it crosses the 0.7s grid line is
the instantaneous velocity of the object at 0.7s.
(B) Incorrect. The acceleration is equal to the slope of the graph of
velocity as a function of time. In this case, since data is being
analyzed, the best fit curve must be determined, and then the
acceleration can be determined from the slope of a line tangent to
that curve at 0.7s.
(C) Incorrect. The area under a curve of velocity as a function of time
represents the displacement or change in position of the object, not
the acceleration.
(D) Correct. The acceleration is equal to the slope of the graph of
velocity as a function of time. Since data is being analyzed, a best-fit
curve must be determined, and the acceleration can be calculated
from the slope of a line tangent to that curve at 0.7s.
Question 5

Science Practice Learning Objective Topic

5.1 3.B.3.3 6.1


(A) Incorrect. Frequency is the inverse of the period of oscillation. As
shown by the data, the period of this object is approximately 0.62s.
1 1
Frequency is equal to 𝑓𝑓 = = = 1.6Hz
𝑇𝑇 0.62s

(B) Incorrect. Frequency is the inverse of the period of oscillation. As


shown by the data, the period of this object is approximately 0.62s.
1 1
Frequency is equal to 𝑓𝑓 = = = 1.6Hz
𝑇𝑇 0.62s

(C) Incorrect. Frequency is the inverse of the period of oscillation. As


shown by the data, the period of this object is approximately 0.62s.
1 1
Frequency is equal to 𝑓𝑓 = = = 1.6Hz
𝑇𝑇 0.62s

(D) Correct. Frequency is the inverse of the period of oscillation. As


shown by the data, the period of this object is approximately 0.62s.
1 1
Frequency is equal to 𝑓𝑓 = = = 1.6Hz
𝑇𝑇 0.62s
Question 6

Science Practice Learning Objective Topic

2.2 3.B.2.1 2.6


(A) Incorrect. The component of 𝐹𝐹0 in the horizontal direction is
identical in both scenarios, however, in scenario 1 the student is
applying an additional component of 𝐹𝐹0 downward, which increases
the normal force exerted by the ground on the box, which also
increases the frictional force between the ground and the box. In
scenario 2, the student exerts that same component of 𝐹𝐹0 vertically
upward, which reduces the normal force exerted by the ground on
the box and the frictional force on the box. This means that the
acceleration of the box in scenario 2 will be greater than the
acceleration of the box in scenario 1. The graphs in choice A show
both boxes are accelerating, however the slope of the line
representing the speed of block 1 has a larger slope representing a
larger acceleration. Since the horizontal component of 𝐹𝐹0 is the same
in both scenarios but the force of friction is larger in scenario 1, box
1 cannot have a larger acceleration.
(B) Incorrect. The component of 𝐹𝐹0 in the horizontal direction is
identical in both scenarios, however, in scenario 1 the student is
applying an additional component of 𝐹𝐹0 downward, which increases
the normal force exerted by the ground on the box, which also
increases the frictional force between the ground and the box. In
scenario 2, the student exerts that same component of 𝐹𝐹0 vertically
upward, which reduces the normal force exerted by the ground on
the box and the frictional force on the box. This means that the
acceleration of the box in scenario 2 will be greater than the
acceleration of the box in scenario 1. The graphs in choice B show
that the velocities of both boxes remain constant as a function of
time, which would not happen in this case. If the forces were
balanced in scenario 1, they cannot be balanced in scenario 2 where
the horizontal component of 𝐹𝐹0 remains constant while the frictional
force decreases.
Question 6 (continued)

(C) Correct. The component of 𝐹𝐹0 in the horizontal direction is identical


in both scenarios, however, in scenario 1 the student is applying an
additional component of 𝐹𝐹0 downward, which increases the normal
force exerted by the ground on the box, which also increases the
frictional force between the ground and the box. In scenario 2, the
student exerts that same component of 𝐹𝐹0 vertically upward, which
reduces the normal force exerted by the ground on the box and the
frictional force on the box. The acceleration of the box in scenario 2
will be greater than the acceleration of the box in scenario 1. The
magnitude of the acceleration of the box is represented by the slope
of the speed vs. time graphs.
The graphs in choice C show that acceleration in scenario 2 is greater
than the acceleration of the box in scenario 1 and that the box
remains at a constant speed in scenario 1. For the box to move with
constant speed in scenario 1, the horizontal components of the forces
must sum to zero. If the forces were balanced in scenario 1, they
cannot be balanced in scenario 2 where the horizontal component of
𝐹𝐹0 remains constant while the frictional force decreases. So the box
in scenario 2 will have a non-zero acceleration.
(D) Incorrect. The component of 𝐹𝐹0 in the horizontal direction is
identical in both scenarios, however, in scenario 1 the student is
applying an additional component of 𝐹𝐹0 downward, which increases
the normal force exerted by the ground on the box, which also
increases the frictional force between the ground and the box. In
scenario 2, the student exerts that same component of 𝐹𝐹0 vertically
upward, which reduces the normal force exerted by the ground on
the box and the frictional force on the box. This means that the
acceleration of the box in scenario 2 will be greater than the
acceleration of the box in scenario 1. The graphs in choice D show
that the velocities of both boxes remain constant as a function of
time, which would not happen in this case. If the forces were
balanced in scenario 1, they cannot be balanced in scenario 2 where
the horizontal component of 𝐹𝐹0 remains constant while the frictional
force decreases.
Question 7

Science Practice Learning Objective Topic

2.2 3.B.1.3 2.6


(A) Correct. The acceleration of the box can be determined by using
Newton’s second law analysis in vertical and horizontal components:
𝛴𝛴𝐹𝐹𝑥𝑥
𝑎𝑎𝑥𝑥 =
𝑀𝑀0
(𝐹𝐹0 cos𝜃𝜃) − 𝐹𝐹𝑓𝑓
𝑎𝑎𝑥𝑥 =
𝑀𝑀0
(𝐹𝐹0 cos𝜃𝜃) − 𝜇𝜇(𝐹𝐹𝑁𝑁 )
𝑎𝑎𝑥𝑥 =
𝑀𝑀0
𝛴𝛴𝐹𝐹𝑦𝑦
𝑎𝑎𝑦𝑦 =
𝑀𝑀0
𝐹𝐹𝑁𝑁 + (𝐹𝐹0 sin𝜃𝜃) − 𝐹𝐹𝑔𝑔
0=
𝑀𝑀0
(𝐹𝐹
𝐹𝐹𝑁𝑁 = 𝐹𝐹𝑔𝑔 − 0 sin𝜃𝜃)
𝐹𝐹𝑁𝑁 = 𝑀𝑀0 𝑔𝑔 − (𝐹𝐹0 sin𝜃𝜃)
(𝐹𝐹0 cos𝜃𝜃) − 𝜇𝜇�𝑀𝑀0 𝑔𝑔 − (𝐹𝐹0 sin𝜃𝜃)�
∴ 𝑎𝑎𝑥𝑥 =
𝑀𝑀0
(B) Incorrect. Using Newton’s second law:
𝛴𝛴𝐹𝐹𝑦𝑦 = 𝐹𝐹𝑁𝑁 + (𝐹𝐹0 sin𝜃𝜃) − 𝐹𝐹𝑔𝑔 = 𝑀𝑀0 𝑎𝑎𝑦𝑦 = 0
𝐹𝐹𝑁𝑁 = 𝐹𝐹𝑔𝑔 − (𝐹𝐹0 sin𝜃𝜃)
𝐹𝐹𝑁𝑁 = 𝑀𝑀0 𝑔𝑔 − (𝐹𝐹0 sin𝜃𝜃)
𝛴𝛴𝐹𝐹𝑥𝑥 = (𝐹𝐹0 cos𝜃𝜃) − 𝐹𝐹𝑓𝑓 = 𝑀𝑀0 𝑎𝑎𝑥𝑥
𝛴𝛴𝐹𝐹𝑥𝑥 = (𝐹𝐹0 cos𝜃𝜃) − 𝜇𝜇(𝐹𝐹𝑁𝑁 ) = 𝑀𝑀0 𝑎𝑎𝑥𝑥
𝛴𝛴𝐹𝐹𝑥𝑥 = (𝐹𝐹0 cos𝜃𝜃) − 𝜇𝜇�𝑀𝑀0 𝑔𝑔 − (𝐹𝐹0 sin𝜃𝜃)� = 𝑀𝑀0 𝑎𝑎𝑥𝑥
(𝐹𝐹0 cos𝜃𝜃) − 𝜇𝜇�𝑀𝑀0 𝑔𝑔 − (𝐹𝐹0 sin𝜃𝜃)�
𝑎𝑎𝑥𝑥 =
𝑀𝑀0
Question 7 (continued)
(C) Incorrect. Using Newton’s second law:
𝛴𝛴𝐹𝐹𝑦𝑦 = 𝐹𝐹𝑁𝑁 + (𝐹𝐹0 sin𝜃𝜃) − 𝐹𝐹𝑔𝑔 = 𝑀𝑀0 𝑎𝑎𝑦𝑦 = 0
𝐹𝐹𝑁𝑁 = 𝐹𝐹𝑔𝑔 − (𝐹𝐹0 sin𝜃𝜃)
𝐹𝐹𝑁𝑁 = 𝑀𝑀0 𝑔𝑔 − (𝐹𝐹0 sin𝜃𝜃)
𝛴𝛴𝐹𝐹𝑥𝑥 = (𝐹𝐹0 cos𝜃𝜃) − 𝐹𝐹𝑓𝑓 = 𝑀𝑀0 𝑎𝑎𝑥𝑥
𝛴𝛴𝐹𝐹𝑥𝑥 = (𝐹𝐹0 cos𝜃𝜃) − 𝜇𝜇(𝐹𝐹𝑁𝑁 ) = 𝑀𝑀0 𝑎𝑎𝑥𝑥
𝛴𝛴𝐹𝐹𝑥𝑥 = (𝐹𝐹0 cos𝜃𝜃) − 𝜇𝜇�𝑀𝑀0 𝑔𝑔 − (𝐹𝐹0 sin𝜃𝜃)� = 𝑀𝑀0 𝑎𝑎𝑥𝑥
(𝐹𝐹0 cos𝜃𝜃) − 𝜇𝜇�𝑀𝑀0 𝑔𝑔 − (𝐹𝐹0 sin𝜃𝜃)�
𝑎𝑎𝑥𝑥 =
𝑀𝑀0
(D) Incorrect. Using Newton’s second law:
𝛴𝛴𝐹𝐹𝑦𝑦 = 𝐹𝐹𝑁𝑁 + (𝐹𝐹0 sin𝜃𝜃) − 𝐹𝐹𝑔𝑔 = 𝑀𝑀0 𝑎𝑎𝑦𝑦 = 0
𝐹𝐹𝑁𝑁 = 𝐹𝐹𝑔𝑔 − (𝐹𝐹0 sin𝜃𝜃)
𝐹𝐹𝑁𝑁 = 𝑀𝑀0 𝑔𝑔 − (𝐹𝐹0 sin𝜃𝜃)
𝛴𝛴𝐹𝐹𝑥𝑥 = (𝐹𝐹0 cos𝜃𝜃) − 𝐹𝐹𝑓𝑓 = 𝑀𝑀0 𝑎𝑎𝑥𝑥
𝛴𝛴𝐹𝐹𝑥𝑥 = (𝐹𝐹0 cos𝜃𝜃) − 𝜇𝜇(𝐹𝐹𝑁𝑁 ) = 𝑀𝑀0 𝑎𝑎𝑥𝑥
𝛴𝛴𝐹𝐹𝑥𝑥 = (𝐹𝐹0 cos𝜃𝜃) − 𝜇𝜇�𝑀𝑀0 𝑔𝑔 − (𝐹𝐹0 sin𝜃𝜃)� = 𝑀𝑀0 𝑎𝑎𝑥𝑥
(𝐹𝐹0 cos𝜃𝜃) − 𝜇𝜇�𝑀𝑀0 𝑔𝑔 − (𝐹𝐹0 sin𝜃𝜃)�
𝑎𝑎𝑥𝑥 =
𝑀𝑀0
Question 8

Science Practice Learning Objective Topic

2.2 4.C.2.2 4.2


(A) Incorrect. Using the work-energy principle, the kinetic energy of the box
at the bottom of the ramp is equal to the gravitational potential energy
of the box-earth system when the box is at the top of the ramp minus the
energy dissipated by friction while the box slides down the ramp.
𝐸𝐸𝑡𝑡𝑡𝑡𝑡𝑡 − 𝐸𝐸𝑑𝑑𝑑𝑑𝑑𝑑𝑑𝑑𝑑𝑑𝑡𝑡𝑑𝑑𝑡𝑡𝑑𝑑𝑑𝑑 = 𝐸𝐸𝑏𝑏𝑡𝑡𝑡𝑡𝑡𝑡𝑡𝑡𝑏𝑏
𝑈𝑈𝑔𝑔 − 𝐹𝐹𝑓𝑓 (𝐿𝐿) = 𝐾𝐾
𝑈𝑈𝑔𝑔 − (𝜇𝜇𝜇𝜇𝑔𝑔cos𝜃𝜃)(𝐿𝐿) = 𝐾𝐾
𝐾𝐾 = 𝜇𝜇𝑔𝑔ℎ − 𝜇𝜇𝜇𝜇𝑔𝑔𝐿𝐿cos𝜃𝜃
(B) Incorrect. Using the work-energy principle, the kinetic energy of the box
at the bottom of the ramp is equal to the gravitational potential energy
of the box-earth system when the box is at the top of the ramp minus the
energy dissipated by friction while the box slides down the ramp.
𝐸𝐸𝑡𝑡𝑡𝑡𝑡𝑡 − 𝐸𝐸𝑑𝑑𝑑𝑑𝑑𝑑𝑑𝑑𝑑𝑑𝑡𝑡𝑑𝑑𝑡𝑡𝑑𝑑𝑑𝑑 = 𝐸𝐸𝑏𝑏𝑡𝑡𝑡𝑡𝑡𝑡𝑡𝑡𝑏𝑏
𝑈𝑈𝑔𝑔 − 𝐹𝐹𝑓𝑓 (𝐿𝐿) = 𝐾𝐾
𝑈𝑈𝑔𝑔 − (𝜇𝜇𝜇𝜇𝑔𝑔cos𝜃𝜃)(𝐿𝐿) = 𝐾𝐾
𝐾𝐾 = 𝜇𝜇𝑔𝑔ℎ − 𝜇𝜇𝜇𝜇𝑔𝑔𝐿𝐿cos𝜃𝜃
(C) Correct. Using the work-energy principle, the kinetic energy of the box
at the bottom of the ramp is equal to the gravitational potential energy
of the box-earth system when the box is at the top of the ramp minus the
energy dissipated by friction while the box slides down the ramp.
𝐸𝐸𝑡𝑡𝑡𝑡𝑡𝑡 − 𝐸𝐸𝑑𝑑𝑑𝑑𝑑𝑑𝑑𝑑𝑑𝑑𝑡𝑡𝑑𝑑𝑡𝑡𝑑𝑑𝑑𝑑 = 𝐸𝐸𝑏𝑏𝑡𝑡𝑡𝑡𝑡𝑡𝑡𝑡𝑏𝑏
𝑈𝑈𝑔𝑔 − 𝐹𝐹𝑓𝑓 (𝐿𝐿) = 𝐾𝐾
𝑈𝑈𝑔𝑔 − (𝜇𝜇𝜇𝜇𝑔𝑔cos𝜃𝜃)(𝐿𝐿) = 𝐾𝐾
𝐾𝐾 = 𝜇𝜇𝑔𝑔ℎ − 𝜇𝜇𝜇𝜇𝑔𝑔𝐿𝐿cos𝜃𝜃
(D) Incorrect. Using the work-energy principle, the kinetic energy of the box
at the bottom of the ramp is equal to the gravitational potential energy
of the box-earth system when the box is at the top of the ramp minus the
energy dissipated by friction while the box slides down the ramp.
𝐸𝐸𝑡𝑡𝑡𝑡𝑡𝑡 − 𝐸𝐸𝑑𝑑𝑑𝑑𝑑𝑑𝑑𝑑𝑑𝑑𝑡𝑡𝑑𝑑𝑡𝑡𝑑𝑑𝑑𝑑 = 𝐸𝐸𝑏𝑏𝑡𝑡𝑡𝑡𝑡𝑡𝑡𝑡𝑏𝑏
𝑈𝑈𝑔𝑔 − 𝐹𝐹𝑓𝑓 (𝐿𝐿) = 𝐾𝐾
𝑈𝑈𝑔𝑔 − (𝜇𝜇𝜇𝜇𝑔𝑔cos𝜃𝜃)(𝐿𝐿) = 𝐾𝐾
𝐾𝐾 = 𝜇𝜇𝑔𝑔ℎ − 𝜇𝜇𝜇𝜇𝑔𝑔𝐿𝐿cos𝜃𝜃
Question 9

Science Practice Learning Objective Topic

5.1 4.B.2.2 5.2


(A) Incorrect. The area under a force as a function of time graph is equal
to the change in the momentum of the object. In this case, the
approximate area under the curve between 0 and 4 seconds is equal to
1
(10N)(2s) + (10N)(2s) = 30Ns
2

(B) Correct. The area under a force as a function of time graph is equal
to the change in the momentum of the object. In this case, the
approximate area under the curve between 0 and 4 seconds is equal to
1
(10N)(2s) + (10N)(2s) = 30Ns
2
(C) Incorrect. The area under a force as a function of time graph is equal
to the change in the momentum of the object. In this case, the
approximate area under the curve between 0 and 4 seconds is equal to
1
(10N)(2s) + (10N)(2s) = 30Ns
2
(D) Incorrect. The area under a force as a function of time graph is equal
to the change in the momentum of the object. In this case, the
approximate area under the curve between 0 and 4 seconds is equal to
1
(10N)(2s) + (10N)(2s) = 30Ns
2
Question 10

Science Practice Learning Objective Topic

1.4 3.B.2.1 2.6


(A) Incorrect. Summing the forces in the vertical direction shows that
the object is not accelerating in the vertical direction.
𝛴𝛴𝐹𝐹𝑦𝑦
𝑎𝑎𝑦𝑦 =
𝜇𝜇
17N + (5sin37°)N − 20N
𝑎𝑎𝑦𝑦 =
(2.0kg)
𝑎𝑎𝑦𝑦 = 0
Summing the forces in the horizontal direction shows that the object
is accelerating in the horizontal direction.
𝛴𝛴𝐹𝐹𝑥𝑥
𝑎𝑎𝑥𝑥 =
𝑏𝑏
(5cos37°)N−3N
𝑎𝑎𝑥𝑥 = (2.0kg)
,
m
𝑎𝑎𝑥𝑥 = �0.5 2 �
s
The object starts from rest, so to find the time it takes the object to
m
accelerate to 2.0 , a kinematic equation can be used.
s
𝑣𝑣𝑓𝑓𝑥𝑥 = 𝑣𝑣𝑑𝑑𝑥𝑥 + 𝑎𝑎𝑥𝑥 𝑡𝑡
m m m
2.0 = 0 + �0.5 2 � 𝑡𝑡
s s s
𝑡𝑡 = 4.0s
(B) Incorrect. Summing the forces in the vertical direction shows that
the object is not accelerating in the vertical direction.
𝛴𝛴𝐹𝐹𝑦𝑦
𝑎𝑎𝑦𝑦 =
𝜇𝜇
17N + (5sin37°)N − 20N
𝑎𝑎𝑦𝑦 =
(2.0kg)
𝑎𝑎𝑦𝑦 = 0
Summing the forces in the horizontal direction shows that the object
is accelerating in the horizontal direction.
𝛴𝛴𝐹𝐹𝑥𝑥
𝑎𝑎𝑥𝑥 =
𝑏𝑏
(5cos37°)N−3N
𝑎𝑎𝑥𝑥 = (2.0kg)
,
m
𝑎𝑎𝑥𝑥 = �0.5 2 �
s
The object starts from rest, so to find the time it takes the object to
m
accelerate to 2.0 , a kinematic equation can be used.
s
𝑣𝑣𝑓𝑓𝑥𝑥 = 𝑣𝑣𝑑𝑑𝑥𝑥 + 𝑎𝑎𝑥𝑥 𝑡𝑡
m m m
2.0 = 0 + �0.5 2 � 𝑡𝑡
s s s
𝑡𝑡 = 4.0s
Question 10 (continued)

(C) Correct. Summing the forces in the vertical direction shows that
the object is not accelerating in the vertical direction.
𝛴𝛴𝐹𝐹𝑦𝑦
𝑎𝑎𝑦𝑦 =
𝜇𝜇
17N + (5sin37°)N − 20N
𝑎𝑎𝑦𝑦 =
(2.0kg)
𝑎𝑎𝑦𝑦 = 0
Summing the forces in the horizontal direction shows that the object
is accelerating in the horizontal direction.
𝛴𝛴𝐹𝐹𝑥𝑥
𝑎𝑎𝑥𝑥 =
𝑏𝑏
(5cos37°)N−3N
𝑎𝑎𝑥𝑥 = (2.0kg)
,
m
𝑎𝑎𝑥𝑥 = �0.5 2 �
s
The object starts from rest, so to find the time it takes the object to
m
accelerate to 2.0 , a kinematic equation can be used.
s
𝑣𝑣𝑓𝑓𝑥𝑥 = 𝑣𝑣𝑑𝑑𝑥𝑥 + 𝑎𝑎𝑥𝑥 𝑡𝑡
m m m
2.0 = 0 + �0.5 2 � 𝑡𝑡
s s s
𝑡𝑡 = 4.0s
(D) Incorrect. Summing the forces in the vertical direction shows that
the object is not accelerating in the vertical direction.
𝛴𝛴𝐹𝐹𝑦𝑦
𝑎𝑎𝑦𝑦 =
𝜇𝜇
17N + (5sin37°)N − 20N
𝑎𝑎𝑦𝑦 =
(2.0kg)
𝑎𝑎𝑦𝑦 = 0
Summing the forces in the horizontal direction shows that the object
is accelerating in the horizontal direction.
𝛴𝛴𝐹𝐹𝑥𝑥
𝑎𝑎𝑥𝑥 =
𝑏𝑏
(5cos37°)N−3N
𝑎𝑎𝑥𝑥 = (2.0kg)
,
m
𝑎𝑎𝑥𝑥 = �0.5 2 �
s
The object starts from rest, so to find the time it takes the object to
m
accelerate to 2.0 , a kinematic equation can be used.
s
𝑣𝑣𝑓𝑓𝑥𝑥 = 𝑣𝑣𝑑𝑑𝑥𝑥 + 𝑎𝑎𝑥𝑥 𝑡𝑡
m m m
2.0 = 0 + �0.5 2 � 𝑡𝑡
s s s
𝑡𝑡 = 4.0s
Question 11

Science Practice Learning Objective Topic

6.4 5.B.4.1 4.3


(A) Incorrect. No outside forces are doing work on the two-star system,
so the mechanical energy of the system is constant. The stars have
the greatest speed when their kinetic energies are greatest, which will
occur when the gravitation potential energy of the system is at a
minimum, which is when the stars are closest together.
(B) Correct. No outside forces are doing work on the two-star system, so
the mechanical energy of the system is constant. The stars have the
greatest speed when their kinetic energies are greatest, which will
occur when the gravitation potential energy of the system is at a
minimum, which is when the stars are closest together.
(C) Incorrect. No outside forces are doing work on the two-star system,
so the mechanical energy of the system is constant. The stars have
the greatest speed when their kinetic energies are greatest, which will
occur when the gravitation potential energy of the system is at a
minimum, which is when the stars are closest together.
(D) Incorrect. No outside forces are doing work on the two-star system,
so the mechanical energy of the system is constant. The stars have
the greatest speed when their kinetic energies are greatest, which will
occur when the gravitation potential energy of the system is at a
minimum, which is when the stars are closest together.
Question 12

Science Practice Learning Objective Topic

6.4 5.B.5.4 4.3


(A) Incorrect. As the cylinder rolls down the ramp, the distance between
the center of earth and the cylinder decreases, causing the potential
energy of the cylinder-Earth system to decrease. Analyzing the
system of only the cylinder, the earth does work on the cylinder as it
rolls down the ramp, increasing the kinetic energy of the cylinder.
(B) Incorrect. As the cylinder rolls down the ramp, the distance between
the center of earth and the cylinder decreases, causing the potential
energy of the cylinder-Earth system to decrease. Analyzing the
system of only the cylinder, the earth does work on the cylinder as it
rolls down the ramp, increasing the kinetic energy of the cylinder.
(C) Correct. As the cylinder rolls down the ramp, the distance between
the center of earth and the cylinder decreases, causing the potential
energy of the cylinder-Earth system to decrease. Analyzing the
system of only the cylinder, the earth does work on the cylinder as it
rolls down the ramp, increasing the kinetic energy of the cylinder.
(D) Incorrect. As the cylinder rolls down the ramp, the distance between
the center of earth and the cylinder decreases, causing the potential
energy of the cylinder-Earth system to decrease. Analyzing the
system of only the cylinder, the earth does work on the cylinder as it
rolls down the ramp, increasing the kinetic energy of the cylinder.
Question 13

Science Practice Learning Objective Topic

6.4 3.E.1.1 4.2


(A) Incorrect. While the cylinder is in the air, the force of gravity is
continuing to do work on the cylinder, causing the translational
kinetic energy to increase, however there is no torque exerted on the
cylinder, and so the rotational kinetic energy of the cylinder stays the
same while it is falling.
(B) Incorrect. While the cylinder is in the air, the force of gravity is
continuing to do work on the cylinder, causing the translational
kinetic energy to increase, however there is no torque exerted on the
cylinder, and so the rotational kinetic energy of the cylinder stays the
same while it is falling.
(C) Correct. While the cylinder is in the air, the force of gravity is
continuing to do work on the cylinder, causing the translational
kinetic energy to increase, however there is no torque exerted on the
cylinder, and so the rotational kinetic energy of the cylinder stays the
same while it is falling.
(D) Incorrect. While the cylinder is in the air, the force of gravity is
continuing to do work on the cylinder, causing the translational
kinetic energy to increase, however there is no torque exerted on the
cylinder, and so the rotational kinetic energy of the cylinder stays the
same while it is falling.
Question 14

Science Practice Learning Objective Topic

6.4 5.A.2.1 4.1


5.B.4.2
(A) Incorrect. While the ratios of translational kinetic energy to
rotational kinetic energy will be different for the sphere and cylinder,
the total kinetic energy of both the sphere and cylinder will be equal
to the total potential energy of the sphere-Earth, or cylinder-Earth
system at the top of the ramp. The sphere and cylinder have the same
mass and are displaced the same vertical distance, so their kinetic
energies at the bottom of the ramp will be equal.
(B) Incorrect. While the ratios of translational kinetic energy to
rotational kinetic energy will be different for the sphere and cylinder,
the total kinetic energy of both the sphere and cylinder will be equal
to the total potential energy of the sphere-Earth, or cylinder-Earth
system at the top of the ramp. The sphere and cylinder have the same
mass and are displaced the same vertical distance, so their kinetic
energies at the bottom of the ramp will be equal.
(C) Incorrect. While the ratios of translational kinetic energy to
rotational kinetic energy will be different for the sphere and cylinder,
the total kinetic energy of both the sphere and cylinder will be equal
to the total potential energy of the sphere-Earth, or cylinder-Earth
system at the top of the ramp. The sphere and cylinder have the same
mass and are displaced the same vertical distance, so their kinetic
energies at the bottom of the ramp will be equal.
(D) Correct. While the ratios of translational kinetic energy to rotational
kinetic energy will be different for the sphere and cylinder, the total
kinetic energy of both the sphere and cylinder will be equal to the
total potential energy of the sphere-Earth, or cylinder-Earth system
at the top of the ramp. The sphere and cylinder have the same mass
and are displaced the same vertical distance, so their kinetic energies
at the bottom of the ramp will be equal.
Question 15

Science Practice Learning Objective Topic

1.4 3.B.2.1 3.7


(A) Incorrect. The circular motion of the ball is a result of the two forces
acting on the ball: the force of gravity exerted on the ball and the
force exerted by the rod on the ball. The radial component of the
vector sum of those forces causes the centripetal acceleration of the
ball. The radius of the circular path and the speed of the ball is
constant, so the centripetal acceleration is constant. The force
exerted by the rod on the ball is always directed towards the center of
the circle, and the force of gravity is always directed vertically
downwards. When the ball is near the top of the circle, the force of
gravity contributes to the centripetal acceleration of the ball, and the
force of the rod on the ball decreases, resulting in a centripetal
acceleration. When the ball is near the bottom of the circle, the rod
must support the weight of the ball and as well as cause the ball to
move in a circular path with the same centripetal acceleration,
causing the force of the rod on the ball to increase. At points 2 and 3,
the force of the rod on the ball is equal and the largest of the nearest
points because the points are closest to the bottom of the circular
path. The force of the rod on the ball is equal at these two points
because they are equidistant from the bottom of the circular path.
The force of the rod on the ball at point 1 is the least of the labeled
points because it is the closest point to the top of the circular path.
(B) Correct. The circular motion of the ball is a result of the two forces
acting on the ball: the force of gravity exerted on the ball and the
force exerted by the rod on the ball. The radial component of the
vector sum of those forces causes the centripetal acceleration of the
ball. The radius of the circular path and the speed of the ball is
constant, so the centripetal acceleration is constant. The force
exerted by the rod on the ball is always directed towards the center of
the circle, and the force of gravity is always directed vertically
downwards. When the ball is near the top of the circle, the force of
gravity contributes to the centripetal acceleration of the ball, and the
force of the rod on the ball decreases, resulting in a centripetal
acceleration. When the ball is near the bottom of the circle, the rod
must support the weight of the ball and as well as cause the ball to
move in a circular path with the same centripetal acceleration,
causing the force of the rod on the ball to increase. At points 2 and 3,
the force of the rod on the ball is equal and the largest of the nearest
points because the points are closest to the bottom of the circular
path. The force of the rod on the ball is equal at these two points
because they are equidistant from the bottom of the circular path.
The force of the rod on the ball at point 1 is the least of the labeled
points because it is the closest point to the top of the circular path.
Question 15 (continued)
(C) Incorrect. The circular motion of the ball is a result of the two forces
acting on the ball: the force of gravity exerted on the ball and the
force exerted by the rod on the ball. The radial component of the
vector sum of those forces causes the centripetal acceleration of the
ball. The radius of the circular path and the speed of the ball is
constant, so the centripetal acceleration is constant. The force
exerted by the rod on the ball is always directed towards the center of
the circle, and the force of gravity is always directed vertically
downwards. When the ball is near the top of the circle, the force of
gravity contributes to the centripetal acceleration of the ball, and the
force of the rod on the ball decreases, resulting in a centripetal
acceleration. When the ball is near the bottom of the circle, the rod
must support the weight of the ball and as well as cause the ball to
move in a circular path with the same centripetal acceleration,
causing the force of the rod on the ball to increase. At points 2 and 3,
the force of the rod on the ball is equal and the largest of the nearest
points because the points are closest to the bottom of the circular
path. The force of the rod on the ball is equal at these two points
because they are equidistant from the bottom of the circular path.
The force of the rod on the ball at point 1 is the least of the labeled
points because it is the closest point to the top of the circular path.
(D) Incorrect. The circular motion of the ball is a result of the two forces
acting on the ball: the force of gravity exerted on the ball and the
force exerted by the rod on the ball. The radial component of the
vector sum of those forces causes the centripetal acceleration of the
ball. The radius of the circular path and the speed of the ball is
constant, so the centripetal acceleration is constant. The force
exerted by the rod on the ball is always directed towards the center of
the circle, and the force of gravity is always directed vertically
downwards. When the ball is near the top of the circle, the force of
gravity contributes to the centripetal acceleration of the ball, and the
force of the rod on the ball decreases, resulting in a centripetal
acceleration. When the ball is near the bottom of the circle, the rod
must support the weight of the ball and as well as cause the ball to
move in a circular path with the same centripetal acceleration,
causing the force of the rod on the ball to increase. At points 2 and 3,
the force of the rod on the ball is equal and the largest of the nearest
points because the points are closest to the bottom of the circular
path. The force of the rod on the ball is equal at these two points
because they are equidistant from the bottom of the circular path.
The force of the rod on the ball at point 1 is the least of the labeled
points because it is the closest point to the top of the circular path.
Question 16

Science Practice Learning Objective Topic

1.4 3.A.1.1 2.6


3.B.1.3
3.B.2.1
(A) Incorrect. At the top of the loop the sum of the force is equal to the
sum of the gravitational force and the normal force exerted by the
loop. In this case, the normal force is not zero since the problem
states that the block has more than enough speed to remain firmly in
contact with the track as it goes around the loop. Because the net

greater than 𝑔𝑔.


force is greater than the weight of the block, the acceleration will be

(B) Incorrect. At the top of the loop the sum of the force is equal to the
sum of the gravitational force and the normal force exerted by the
loop. In this case, the normal force is not zero since the problem
states that the block has more than enough speed to remain firmly in
contact with the track as it goes around the loop. Because the net

greater than 𝑔𝑔.


force is greater than the weight of the block, the acceleration will be

(C) Incorrect. At the top of the loop the sum of the force is equal to the
sum of the gravitational force and the normal force exerted by the
loop. In this case, the normal force is not zero since the problem
states that the block has more than enough speed to remain firmly in
contact with the track as it goes around the loop. Because the net

greater than 𝑔𝑔.


force is greater than the weight of the block, the acceleration will be

(D) Correct. At the top of the loop the sum of the force is equal to the
sum of the gravitational force and the normal force exerted by the
loop. In this case, the normal force is not zero since the problem
states that the block has more than enough speed to remain firmly in
contact with the track as it goes around the loop. Because the net

greater than 𝑔𝑔.


force is greater than the weight of the block, the acceleration will be
Question 17

Science Practice Learning Objective Topic

1.4 5.B.4.2 4.3


(A) Incorrect. The mechanical energy of the bob-Earth system is constant,
so the mechanical energy of the bob-Earth system when the bob is
1.0 m above the surface is equal to the mechanical energy of the
bob-Earth system when the bob is 0.8 m above the surface of Earth.
𝐸𝐸𝑑𝑑 = 𝐸𝐸𝑓𝑓
𝑈𝑈𝑔𝑔𝑖𝑖 = 𝐾𝐾 + 𝑈𝑈𝑔𝑔𝑓𝑓
1
𝜇𝜇𝑔𝑔ℎ𝑑𝑑 = 𝜇𝜇𝑣𝑣𝑓𝑓2 + 𝜇𝜇𝑔𝑔ℎ𝑓𝑓
2
1
𝜇𝜇𝑔𝑔ℎ𝑑𝑑 − 𝜇𝜇𝑔𝑔ℎ𝑓𝑓 = 𝜇𝜇𝑣𝑣𝑓𝑓2
2
1 2
𝑔𝑔�ℎ𝑑𝑑 − ℎ𝑓𝑓 � = 𝑣𝑣
2 𝑓𝑓

𝑣𝑣𝑓𝑓 = �2𝑔𝑔�ℎ𝑑𝑑 − ℎ𝑓𝑓 �

m
𝑣𝑣𝑓𝑓 = �2 �10 2 � (1.0m − 0.8m)
s
m
𝑣𝑣𝑓𝑓 = 2.0
s
(B) Incorrect. The mechanical energy of the bob-Earth system is constant,
so the mechanical energy of the bob-Earth system when the bob is
1.0 m above the surface is equal to the mechanical energy of the
bob-Earth system when the bob is 0.8 m above the surface of Earth.
𝐸𝐸𝑑𝑑 = 𝐸𝐸𝑓𝑓
𝑈𝑈𝑔𝑔𝑖𝑖 = 𝐾𝐾 + 𝑈𝑈𝑔𝑔𝑓𝑓
1
𝜇𝜇𝑔𝑔ℎ𝑑𝑑 = 𝜇𝜇𝑣𝑣𝑓𝑓2 + 𝜇𝜇𝑔𝑔ℎ𝑓𝑓
2
1
𝜇𝜇𝑔𝑔ℎ𝑑𝑑 − 𝜇𝜇𝑔𝑔ℎ𝑓𝑓 = 𝜇𝜇𝑣𝑣𝑓𝑓2
2
1 2
𝑔𝑔�ℎ𝑑𝑑 − ℎ𝑓𝑓 � = 𝑣𝑣𝑓𝑓
2
𝑣𝑣𝑓𝑓 = �2𝑔𝑔�ℎ𝑑𝑑 − ℎ𝑓𝑓 �
m
𝑣𝑣𝑓𝑓 = �2 �10 2 � (1.0m − 0.8m)
s
m
𝑣𝑣𝑓𝑓 = 2.0
s
Question 17 (continued)

(C) Correct. The mechanical energy of the bob-Earth system is constant,


so the mechanical energy of the bob-Earth system when the bob is
1.0 m above the surface is equal to the mechanical energy of the
bob-Earth system when the bob is 0.8 m above the surface of Earth.
𝐸𝐸𝑑𝑑 = 𝐸𝐸𝑓𝑓
𝑈𝑈𝑔𝑔𝑖𝑖 = 𝐾𝐾 + 𝑈𝑈𝑔𝑔𝑓𝑓
1
𝜇𝜇𝑔𝑔ℎ𝑑𝑑 = 𝜇𝜇𝑣𝑣𝑓𝑓2 + 𝜇𝜇𝑔𝑔ℎ𝑓𝑓
2
1
𝜇𝜇𝑔𝑔ℎ𝑑𝑑 − 𝜇𝜇𝑔𝑔ℎ𝑓𝑓 = 𝜇𝜇𝑣𝑣𝑓𝑓2
2
1 2
𝑔𝑔�ℎ𝑑𝑑 − ℎ𝑓𝑓 � = 𝑣𝑣𝑓𝑓
2
𝑣𝑣𝑓𝑓 = �2𝑔𝑔�ℎ𝑑𝑑 − ℎ𝑓𝑓 �
m
𝑣𝑣𝑓𝑓 = �2 �10 2 � (1.0m − 0.8m)
s
m
𝑣𝑣𝑓𝑓 = 2.0
s
(D) Incorrect. The mechanical energy of the bob-Earth system is constant,
so the mechanical energy of the bob-Earth system when the bob is
1.0 m above the surface is equal to the mechanical energy of the
bob-Earth system when the bob is 0.8 m above the surface of Earth.
𝐸𝐸𝑑𝑑 = 𝐸𝐸𝑓𝑓
𝑈𝑈𝑔𝑔𝑖𝑖 = 𝐾𝐾 + 𝑈𝑈𝑔𝑔𝑓𝑓
1
𝜇𝜇𝑔𝑔ℎ𝑑𝑑 = 𝜇𝜇𝑣𝑣𝑓𝑓2 + 𝜇𝜇𝑔𝑔ℎ𝑓𝑓
2
1
𝜇𝜇𝑔𝑔ℎ𝑑𝑑 − 𝜇𝜇𝑔𝑔ℎ𝑓𝑓 = 𝜇𝜇𝑣𝑣𝑓𝑓2
2
1 2
𝑔𝑔�ℎ𝑑𝑑 − ℎ𝑓𝑓 � = 𝑣𝑣𝑓𝑓
2
𝑣𝑣𝑓𝑓 = �2𝑔𝑔�ℎ𝑑𝑑 − ℎ𝑓𝑓 �
m
𝑣𝑣𝑓𝑓 = �2 �10 2 � (1.0m − 0.8m)
s
m
𝑣𝑣𝑓𝑓 = 2.0
s
Question 18

Science Practice Learning Objective Topic

7.2 3.E.1.1 4.2


4.C.1.2
5.B.3.2
5.B.5.5
(A) Correct. Since there are two additional external forces doing work
on the system, whether or not the mechanical energy of the system
increases, decreases or remains the same depends on the net work
done by the additional external forces. The two forces are equal in
magnitude but in different directions, and so even though the
displacement of the box is the same, the work done by the two forces
is different. The work done by each force can be determined:
𝑊𝑊 = 𝐹𝐹𝐹𝐹cos𝜃𝜃
𝑊𝑊𝐹𝐹-vertical = (𝐹𝐹𝐿𝐿cos120°)
−𝐹𝐹𝐹𝐹
𝑊𝑊𝐹𝐹-vertical =
2
𝑊𝑊𝐹𝐹-horizontal = (𝐹𝐹𝐿𝐿cos30°)
√3𝐹𝐹𝐹𝐹
𝑊𝑊𝐹𝐹-horizontal = � �
2
Because the positive work is greater than the negative work, the total
work done on the system is positive and so the mechanical energy of
the block-Earth system increased.
(B) Incorrect. Since there are two additional external forces doing work
on the system, whether or not the mechanical energy of the system
increases, decreases or remains the same depends on the net work
done by the additional external forces. The two forces are equal in
magnitude but in different directions, and so even though the
displacement of the box is the same, the work done by the two forces
is different. The work done by each force can be determined:
𝑊𝑊 = 𝐹𝐹𝐹𝐹cos𝜃𝜃
𝑊𝑊𝐹𝐹-vertical = (𝐹𝐹𝐿𝐿cos120°)
−𝐹𝐹𝐹𝐹
𝑊𝑊𝐹𝐹-vertical =
2
𝑊𝑊𝐹𝐹-horizontal = (𝐹𝐹𝐿𝐿cos30°)
√3𝐹𝐹𝐹𝐹
𝑊𝑊𝐹𝐹-horizontal = � �
2
Because the positive work is greater than the negative work, the total
work done on the system is positive and so the mechanical energy of
the block-Earth system increased.
Question 18 (continued)
(C) Incorrect. Since there are two additional external forces doing work
on the system, whether or not the mechanical energy of the system
increases, decreases or remains the same depends on the net work
done by the additional external forces. The two forces are equal in
magnitude but in different directions, and so even though the
displacement of the box is the same, the work done by the two forces
is different. The work done by each force can be determined:
𝑊𝑊 = 𝐹𝐹𝐹𝐹cos𝜃𝜃
𝑊𝑊𝐹𝐹-vertical = (𝐹𝐹𝐿𝐿cos120°)
−𝐹𝐹𝐹𝐹
𝑊𝑊𝐹𝐹-vertical =
2
𝑊𝑊𝐹𝐹-horizontal = (𝐹𝐹𝐿𝐿cos30°)
√3𝐹𝐹𝐹𝐹
𝑊𝑊𝐹𝐹-horizontal = � �
2
Because the positive work is greater than the negative work, the total
work done on the system is positive and so the mechanical energy of
the block-Earth system increased.
(D) Incorrect. Since there are two additional external forces doing work
on the system, whether or not the mechanical energy of the system
increases, decreases or remains the same depends on the net work
done by the additional external forces. The two forces are equal in
magnitude but in different directions, and so even though the
displacement of the box is the same, the work done by the two forces
is different. The work done by each force can be determined:
𝑊𝑊 = 𝐹𝐹𝐹𝐹cos𝜃𝜃
𝑊𝑊𝐹𝐹-vertical = (𝐹𝐹𝐿𝐿cos120°)
−𝐹𝐹𝐹𝐹
𝑊𝑊𝐹𝐹-vertical =
2
𝑊𝑊𝐹𝐹-horizontal = (𝐹𝐹𝐿𝐿cos30°)
√3𝐹𝐹𝐹𝐹
𝑊𝑊𝐹𝐹-horizontal = � �
2
Because the positive work is greater than the negative work, the total
work done on the system is positive and so the mechanical energy of
the block-Earth system increased.
Question 19

Science Practice Learning Objective Topic

1.4 4.C.1.1 4.2


4.C.1.2
5.B.3.2
5.B.5.5
(A) Incorrect. The energy dissipated due to air resistance is equal to the
change in the mechanical energy of the apple-earth system. Initially,
the system only has gravitational potential energy, and just before the
apple hits the ground, the mechanical energy of the system consists
of only the kinetic energy of the apple. The energy dissipated divided
by the initial gravitational potential energy of the system leads to a
calculation of the percent of energy dissipated due to air resistance.
m
𝐸𝐸𝑑𝑑 = 𝜇𝜇𝑔𝑔ℎ = (𝜇𝜇) �10 2 � (500m)
s
1 1 m 2
𝐸𝐸𝑓𝑓 = 𝜇𝜇𝑣𝑣𝑓𝑓2 = 𝜇𝜇 �40 �
2 2 s
m2 m2
|𝛥𝛥𝐸𝐸| = �𝐸𝐸𝑓𝑓 − 𝐸𝐸𝑑𝑑 � = ��800 � 𝜇𝜇 − �5000 � 𝜇𝜇�
s2 s2
m2
|𝛥𝛥𝐸𝐸| = �4200 � 𝜇𝜇
s2
m2
�4200 2 �𝑏𝑏
s
%Dissipated = m2
⋅ 100% = 84%
�5000 2 �𝑏𝑏
s

(B) Incorrect. The energy dissipated due to air resistance is equal to the
change in the mechanical energy of the apple-earth system. Initially,
the system only has gravitational potential energy, and just before the
apple hits the ground, the mechanical energy of the system consists
of only the kinetic energy of the apple. The energy dissipated divided
by the initial gravitational potential energy of the system leads to a
calculation of the percent of energy dissipated due to air resistance.
m
𝐸𝐸𝑑𝑑 = 𝜇𝜇𝑔𝑔ℎ = (𝜇𝜇) �10 2 � (500m)
s
1 1 m 2
𝐸𝐸𝑓𝑓 = 𝜇𝜇𝑣𝑣𝑓𝑓2 = 𝜇𝜇 �40 �
2 2 s
m2 m2
|𝛥𝛥𝐸𝐸| = �𝐸𝐸𝑓𝑓 − 𝐸𝐸𝑑𝑑 � = ��800 � 𝜇𝜇 − �5000 � 𝜇𝜇�
s2 s2
m2
|𝛥𝛥𝐸𝐸| = �4200 � 𝜇𝜇
s2
m2
�4200 2 �𝑏𝑏
s
%Dissipated = m2
⋅ 100% = 84%
�5000 2 �𝑏𝑏
s
Question 19 (continued)
(C) Incorrect. The energy dissipated due to air resistance is equal to the
change in the mechanical energy of the apple-earth system. Initially,
the system only has gravitational potential energy, and just before the
apple hits the ground, the mechanical energy of the system consists
of only the kinetic energy of the apple. The energy dissipated divided
by the initial gravitational potential energy of the system leads to a
calculation of the percent of energy dissipated due to air resistance.
m
𝐸𝐸𝑑𝑑 = 𝜇𝜇𝑔𝑔ℎ = (𝜇𝜇) �10 2 � (500m)
s
1 1 m 2
𝐸𝐸𝑓𝑓 = 𝜇𝜇𝑣𝑣𝑓𝑓2 = 𝜇𝜇 �40 �
2 2 s
m2 m2
|𝛥𝛥𝐸𝐸| = �𝐸𝐸𝑓𝑓 − 𝐸𝐸𝑑𝑑 � = ��800 � 𝜇𝜇 − �5000 � 𝜇𝜇�
s2 s2
m2
|𝛥𝛥𝐸𝐸| = �4200 � 𝜇𝜇
s2
m2
�4200 2 �𝑏𝑏
s
%Dissipated = m2
⋅ 100% = 84%
�5000 2 �𝑏𝑏
s

(D) Correct. The energy dissipated due to air resistance is equal to the
change in the mechanical energy of the apple-earth system. Initially,
the system only has gravitational potential energy, and just before the
apple hits the ground, the mechanical energy of the system consists
of only the kinetic energy of the apple. The energy dissipated divided
by the initial gravitational potential energy of the system leads to a
calculation of the percent of energy dissipated due to air resistance.
m
𝐸𝐸𝑑𝑑 = 𝜇𝜇𝑔𝑔ℎ = (𝜇𝜇) �10 2 � (500m)
s
1 1 m 2
𝐸𝐸𝑓𝑓 = 𝜇𝜇𝑣𝑣𝑓𝑓2 = 𝜇𝜇 �40 �
2 2 s
m2 m2
|𝛥𝛥𝐸𝐸| = �𝐸𝐸𝑓𝑓 − 𝐸𝐸𝑑𝑑 � = ��800 � 𝜇𝜇 − �5000 � 𝜇𝜇�
s2 s2
m2
|𝛥𝛥𝐸𝐸| = �4200 � 𝜇𝜇
s2
m2
�4200 2 �𝑏𝑏
s
%Dissipated = m2
⋅ 100% = 84%
�5000 2 �𝑏𝑏
s
Question 20

Science Practice Learning Objective Topic

1.5 3.A.1.1 1.1


(A) Correct. Rock 1 will travel upwards, and then again return to the
initial height ℎ0 of the cliff before falling to the ground. When Rock
1 reaches ℎ0 , it has the same velocity as rock 2, therefore rocks 1 and
2 will reach the ground with the same vertical component of velocity.
Rock 3 starts with a zero-vertical component of the velocity, and so
the vertical component just before hitting the ground for rock 3 will
be less than the vertical components of the velocities of rocks 1 and 2.
(B) Incorrect. Rock 1 will travel upwards, and then again return to the
initial height ℎ0 of the cliff before falling to the ground. When Rock
1 reaches ℎ0 , it has the same velocity as rock 2, therefore rocks 1 and
2 will reach the ground with the same vertical component of velocity.
Rock 3 starts with a zero-vertical component of the velocity, and so
the vertical component just before hitting the ground for rock 3 will
be less than the vertical components of the velocities of rocks 1 and 2.
(C) Incorrect. Rock 1 will travel upwards, and then again return to the
initial height ℎ0 of the cliff before falling to the ground. When Rock
1 reaches ℎ0 , it has the same velocity as rock 2, therefore rocks 1 and
2 will reach the ground with the same vertical component of velocity.
Rock 3 starts with a zero-vertical component of the velocity, and so
the vertical component just before hitting the ground for rock 3 will
be less than the vertical components of the velocities of rocks 1 and 2.
(D) Incorrect. Rock 1 will travel upwards, and then again return to the
initial height ℎ0 of the cliff before falling to the ground. When Rock
1 reaches ℎ0 , it has the same velocity as rock 2, therefore rocks 1 and
2 will reach the ground with the same vertical component of velocity.
Rock 3 starts with a zero-vertical component of the velocity, and so
the vertical component just before hitting the ground for rock 3 will
be less than the vertical components of the velocities of rocks 1 and 2.
Question 21

Science Practice Learning Objective Topic

1.4 3.E.1.1 4.2


3.E.1.4
4.C.1.1
4.C.2.2
5.B.4.2
(A) Correct. On the way up to the maximum height, the gravitational
force is exerted downward on rock 1, in the opposite direction of the
vertical displacement of the rock. This means that the force of gravity
does negative work on the rock equal to 𝑊𝑊 = −𝜇𝜇𝑔𝑔𝛥𝛥𝑚𝑚. On the way
back down from the maximum height, the force of gravity is still
downwards and in the same direction as the displacement of the
rock. The work done by the gravitational force on the way down is
equal to 𝑊𝑊 = +𝜇𝜇𝑔𝑔𝛥𝛥𝑚𝑚. This means that the net work done by the
gravitational force between the time rock 1 is launched and the time
when it again reaches height ℎ0 is zero. Alternatively, since the rock
will have the same speed as it did at launch, when it reaches height
ℎ0 again, the net work on the rock must have been zero because it
has the same kinetic energy. The only force exerted on the rock is
gravity, so the work done on the rock by gravity is equal to the net
work.
(B) Incorrect. On the way up to the maximum height, the gravitational
force is exerted downward on rock 1, in the opposite direction of the
vertical displacement of the rock. This means that the force of gravity
does negative work on the rock equal to 𝑊𝑊 = −𝜇𝜇𝑔𝑔𝛥𝛥𝑚𝑚. On the way
back down from the maximum height, the force of gravity is still
downwards and in the same direction as the displacement of the
rock. The work done by the gravitational force on the way down is
equal to 𝑊𝑊 = 𝜇𝜇𝑔𝑔𝛥𝛥𝑚𝑚. This means that the net work done by the
gravitational force between the time rock 1 is launched and the time
when it again reaches height ℎ0 is zero. Alternatively, since the rock
will have the same speed as it did at launch, when it reaches height
ℎ0 again, the net work on the rock must have been zero because it
has the same kinetic energy.
Question 21 (continued)
(C) Incorrect. On the way up to the maximum height, the gravitational
force is exerted downward on rock 1, in the opposite direction of the
vertical displacement of the rock. This means that the force of gravity
does negative work on the rock equal to 𝑊𝑊 = −𝜇𝜇𝑔𝑔𝛥𝛥𝑚𝑚. On the way
back down from the maximum height, the force of gravity is still
downwards and in the same direction as the displacement of the
rock. The work done by the gravitational force on the way down is
equal to 𝑊𝑊 = 𝜇𝜇𝑔𝑔𝛥𝛥𝑚𝑚. This means that the net work done by the
gravitational force between the time rock 1 is launched and the time
when it again reaches height ℎ0 is zero. Alternatively, since the rock
will have the same speed as it did at launch, when it reaches height
ℎ0 again, the net work on the rock must have been zero because it
has the same kinetic energy.
(D) Incorrect. On the way up to the maximum height, the gravitational
force is exerted downward on rock 1, in the opposite direction of the
vertical displacement of the rock. This means that the force of gravity
does negative work on the rock equal to 𝑊𝑊 = −𝜇𝜇𝑔𝑔𝛥𝛥𝑚𝑚. On the way
back down from the maximum height, the force of gravity is still
downwards and in the same direction as the displacement of the
rock. The work done by the gravitational force on the way down is
equal to 𝑊𝑊 = 𝜇𝜇𝑔𝑔𝛥𝛥𝑚𝑚. This means that the net work done by the
gravitational force between the time rock 1 is launched and the time
when it again reaches height ℎ0 is zero. Alternatively, since the rock
will have the same speed as it did at launch, when it reaches height
ℎ0 again, the net work on the rock must have been zero because it
has the same kinetic energy.
Question 22

Science Practice Learning Objective Topic

2.2 3.A.1.1 4.3


5.B.5.5
(A) Incorrect. The net force exerted on the marshmallow does work on
the it, changing the kinetic energy of the marshmallow.
𝑊𝑊 = 𝛥𝛥𝐾𝐾
1
𝐹𝐹𝐹𝐹cos𝜃𝜃 = 𝜇𝜇𝑣𝑣 2
2
1
(0.7N)(0.40m) = (0.0025kg)𝑣𝑣 2
2
𝑣𝑣 = 15m/s
(B) Incorrect. The net force exerted on the marshmallow does work on
the it, changing the kinetic energy of the marshmallow.
𝑊𝑊 = 𝛥𝛥𝐾𝐾
1
𝐹𝐹𝐹𝐹cos𝜃𝜃 = 𝜇𝜇𝑣𝑣 2
2
1
(0.7N)(0.40m) = (0.0025kg)𝑣𝑣 2
2
𝑣𝑣 = 15m/s
(C) Correct. The net force exerted on the marshmallow does work on
the it, changing the kinetic energy of the marshmallow.
𝑊𝑊 = 𝛥𝛥𝐾𝐾
1
𝐹𝐹𝐹𝐹cos𝜃𝜃 = 𝜇𝜇𝑣𝑣 2
2
1
(0.7N)(0.40m) = (0.0025kg)𝑣𝑣 2
2
𝑣𝑣 = 15m/s
(D) Incorrect. The net force exerted on the marshmallow does work on
the it, changing the kinetic energy of the marshmallow.
𝑊𝑊 = 𝛥𝛥𝐾𝐾
1
𝐹𝐹𝐹𝐹cos𝜃𝜃 = 𝜇𝜇𝑣𝑣 2
2
1
(0.7N)(0.40m) = (0.0025kg)𝑣𝑣 2
2
𝑣𝑣 = 15m/s
Question 23

Science Practice Learning Objective Topic

1.4 3.B.1.1 2.6


3.B.2.1
4.A.2.1
(A) Incorrect. There are only two forces exerted on the meterstick as it
falls. One is the force of gravity, which is exerted straight down, and
the normal force from the table which is exerted straight up. Since
there are no horizontal forces, the center of mass can only have a
vertical acceleration, and so must fall straight down.
(B) Incorrect. There are only two forces exerted on the meterstick as it
falls. One is the force of gravity, which is exerted straight down, and
the normal force from the table which is exerted straight up. Since
there are no horizontal forces, the center of mass can only have a
vertical acceleration, and so must fall straight down.
(C) Correct. There are only two forces exerted on the meterstick as it
falls. One is the force of gravity, which is exerted straight down, and
the normal force from the table which is exerted straight up. Since
there are no horizontal forces, the center of mass can only have a
vertical acceleration, and so must fall straight down.
(D) Incorrect. There are only two forces exerted on the meterstick as it
falls. One is the force of gravity, which is exerted straight down, and
the normal force from the table which is exerted straight up. Since
there are no horizontal forces, the center of mass can only have a
vertical acceleration, and so must fall straight down.
Question 24

Science Practice Learning Objective Topic

7.2 3.A.3.1 2.5


(A) Incorrect. In trial 2, the force that the spring exerts on block X is
equal and opposite to the force that the spring exerts on block Y.
Therefore, the net force exerted by the spring on the system is zero.
So, the net force exerted on the system in trial 1 and 2 is the same.
However, in trial 2, when analyzing only block Y, there is the
additional force exerted on the block by the spring, so the net force
on block Y is different in trial 2.
(B) Incorrect. In trial 2, the force that the spring exerts on block X is
equal and opposite to the force that the spring exerts on block Y.
Therefore, the net force exerted by the spring on the system is zero.
So, the net force exerted on the system in trial 1 and 2 is the same.
However, in trial 2, when analyzing only block Y, there is the
additional force exerted on the block by the spring, so the net force
on block y is different in trial 2.
(C) Correct. In trial 2, the force that the spring exerts on block X is equal
and opposite to the force that the spring exerts on block Y.
Therefore, the net force exerted by the spring on the system is zero.
So, the net force exerted on the system in trial 1 and 2 is the same.
However, in trial 2, when analyzing only block Y, there is the
additional force exerted on the block by the spring, so the net force
on block Y is different in trial 2.
(D) Incorrect. In trial 2, the force that the spring exerts on block X is
equal and opposite to the force that the spring exerts on block Y.
Therefore, the net force exerted by the spring on the system is zero.
So, the net force exerted on the system in trial 1 and 2 is the same.
However, in trial 2, when analyzing only block Y, there is the
additional force exerted on the block by the spring, so the net force
on block Y is different in trial 2.
Question 25

Science Practice Learning Objective Topic

6.4 5.B.5.4 4.3


(A) Incorrect. The mechanical energy dissipated by the force of friction
during each interval is proportional to the magnitude of the
frictional force as well as the displacement of the block during each
interval. In interval 1, there is no friction and the velocity of the box
3𝑣𝑣0 −2𝑣𝑣0
increases, and so the block is accelerating at a rate of ( ).
𝑡𝑡0
During time interval 3, the acceleration is equal in magnitude to the
acceleration in interval 1, but opposite in direction. This must mean
that the force of friction in interval 3 is twice the component of the
gravitational force exerted on the block down the ramp. During
intervals 2 and 4, the block travels at a constant velocity, the
acceleration of the block is zero, and so the force of friction must be
equal to the component of the gravitational force exerted on the
block down the ramp. The displacement of the block during each
segment is equal to the area bounded by the velocity as a function of
time graph. Therefore, the mechanical energy dissipated by the
frictional force in each segment can be compared as follows:
Segments 2 & 4 – the force of friction is the same, but the area under
the curve is greater for segment 2, therefore, more energy is
dissipated by friction in segment 2. Segment 3 – the force of friction
is twice as large as it is in segments 2 and 4, and the area under the
5
curve is only as much displacement. Therefore, the mechanical
6
energy dissipated by friction in segment 3 is larger than the
mechanical energy dissipated in either segment 2 or 4.
Question 25 (continued)

(B) Correct. The mechanical energy dissipated by the force of friction


during each interval is proportional to the magnitude of the
frictional force as well as the displacement of the block during each
interval. In interval 1, there is no friction and the velocity of the box
3𝑣𝑣0 −2𝑣𝑣0
increases, and so the block is accelerating at a rate of ( ).
𝑡𝑡0
During time interval 3, the acceleration is equal in magnitude to the
acceleration in interval 1, but opposite in direction. This must mean
that the force of friction in interval 3 is twice the component of the
gravitational force exerted on the block down the ramp. During
intervals 2 and 4, the block travels at a constant velocity, the
acceleration of the block is zero, and so the force of friction must be
equal to the component of the gravitational force exerted on the
block down the ramp. The displacement of the block during each
segment is equal to the area bounded by the velocity as a function of
time graph. Therefore, the mechanical energy dissipated by the
frictional force in each segment can be compared as follows:
Segments 2 & 4 – the force of friction is the same, but the area under
the curve is greater for segment 2, therefore, more energy is
dissipated by friction in segment 2. Segment 3 – the force of friction
is twice as large as it is in segments 2 and 4, and the area under the
5
curve is only as much displacement. Therefore, the mechanical
6
energy dissipated by friction in segment 3 is larger than the
mechanical energy dissipated in either segment 2 or 4.
Question 25 (continued)
(C) Incorrect. The mechanical energy dissipated by the force of friction
during each interval is proportional to the magnitude of the
frictional force as well as the displacement of the block during each
interval. In interval 1, there is no friction and the velocity of the box
3𝑣𝑣0 −2𝑣𝑣0
increases, and so the block is accelerating at a rate of ( ).
𝑡𝑡0
During time interval 3, the acceleration is equal in magnitude to the
acceleration in interval 1, but opposite in direction. This must mean
that the force of friction in interval 3 is twice the component of the
gravitational force exerted on the block down the ramp. During
intervals 2 and 4, the block travels at a constant velocity, the
acceleration of the block is zero, and so the force of friction must be
equal to the component of the gravitational force exerted on the
block down the ramp. The displacement of the block during each
segment is equal to the area bounded by the velocity as a function of
time graph. Therefore, the mechanical energy dissipated by the
frictional force in each segment can be compared as follows:
Segments 2 & 4 – the force of friction is the same, but the area under
the curve is greater for segment 2, therefore, more energy is
dissipated by friction in segment 2. Segment 3 – the force of friction
is twice as large as it is in segments 2 and 4, and the area under the
5
curve is only as much displacement. Therefore, the mechanical
6
energy dissipated by friction in segment 3 is larger than the
mechanical energy dissipated in either segment 2 or 4.
Question 25 (continued)
(D) Incorrect. The mechanical energy dissipated by the force of friction
during each interval is proportional to the magnitude of the
frictional force as well as the displacement of the block during each
interval. In interval 1, there is no friction and the velocity of the box
3𝑣𝑣0 −2𝑣𝑣0
increases, and so the block is accelerating at a rate of ( ).
𝑡𝑡0
During time interval 3, the acceleration is equal in magnitude to the
acceleration in interval 1, but opposite in direction. This must mean
that the force of friction in interval 3 is twice the component of the
gravitational force exerted on the block down the ramp. During
intervals 2 and 4, the block travels at a constant velocity, the
acceleration of the block is zero, and so the force of friction must be
equal to the component of the gravitational force exerted on the
block down the ramp. The displacement of the block during each
segment is equal to the area bounded by the velocity as a function of
time graph. Therefore, the mechanical energy dissipated by the
frictional force in each segment can be compared as follows:
Segments 2 & 4 – the force of friction is the same, but the area under
the curve is greater for segment 2, therefore, more energy is
dissipated by friction in segment 2. Segment 3 – the force of friction
is twice as large as it is in segments 2 and 4, and the area under the
5
curve is only as much displacement. Therefore, the mechanical
6
energy dissipated by friction in segment 3 is larger than the
mechanical energy dissipated in either segment 2 or 4.
Question 26

Science Practice Learning Objective Topic

7.2 5.D.1.1 5.4


(A) Correct. Since the objects collide elastically, the kinetic energy of the
two-object system before the collision is equal to the kinetic energy
of the two-object system after the collision. Since the kinetic energy
of object X decreases, the kinetic energy of object Y must increase.
Since the mass of object Y remains constant, if the kinetic energy of
object Y increases, the speed of object Y must also increase.
(B) Incorrect. Since the objects collide elastically, the kinetic energy of
the two-object system before the collision is equal to the kinetic
energy of the two object system after the collision. Since the kinetic
energy of object X decreases, the kinetic energy of object Y must
increase. Since the mass of object Y remains constant, if the kinetic
energy of object Y increases, the speed of object Y must also increase.
(C) Incorrect. Since the objects collide elastically, the kinetic energy of
the two-object system before the collision is equal to the kinetic
energy of the two object system after the collision. Since the kinetic
energy of object X decreases, the kinetic energy of object Y must
increase. Since the mass of object Y remains constant, if the kinetic
energy of object Y increases, the speed of object Y must also increase.
(D) Incorrect. Since the objects collide elastically, the kinetic energy of
the two-object system before the collision is equal to the kinetic
energy of the two object system after the collision. Since the kinetic
energy of object X decreases, the kinetic energy of object Y must
increase. Since the mass of object Y remains constant, if the kinetic
energy of object Y increases, the speed of object Y must also increase.
Question 27

Science Practice Learning Objective Topic

1.4 3.F.2.1 7.3


4.D.1.1
(A) Incorrect. The change in angular momentum is equal to the net
torque exerted on the object times the time the torque is exerted.
(𝛴𝛴𝜏𝜏𝑑𝑑𝑥𝑥𝑡𝑡 )𝛥𝛥𝑡𝑡 = 𝛥𝛥𝐿𝐿 The rate of change of angular momentum is then
𝛥𝛥𝐹𝐹
equal to the net torque. (𝛴𝛴𝜏𝜏𝑑𝑑𝑥𝑥𝑡𝑡 ) = The torque exerted by each
𝛥𝛥𝑡𝑡
force is equal to the magnitude of the force multiplied by the distance
between the application of the force and the pivot point 𝛴𝛴𝜏𝜏 = 𝐹𝐹𝐹𝐹
The largest torque is given by the 60N force as it is three times
further from the pivot as the 150N force.
(B) Incorrect. The change in angular momentum is equal to the net
torque exerted on the object times the time the torque is exerted.
(𝛴𝛴𝜏𝜏𝑑𝑑𝑥𝑥𝑡𝑡 )𝛥𝛥𝑡𝑡 = 𝛥𝛥𝐿𝐿 The rate of change of angular momentum is then
𝛥𝛥𝐹𝐹
equal to the net torque. (𝛴𝛴𝜏𝜏𝑑𝑑𝑥𝑥𝑡𝑡 ) = The torque exerted by each
𝛥𝛥𝑡𝑡
force is equal to the magnitude of the force multiplied by the distance
between the application of the force and the pivot point 𝛴𝛴𝜏𝜏 = 𝐹𝐹𝐹𝐹
The largest torque is given by the 60N force as it is three times
further from the pivot as the 150N force.
(C) Correct. The change in angular momentum is equal to the net
torque exerted on the object times the time the torque is exerted.
(𝛴𝛴𝜏𝜏𝑑𝑑𝑥𝑥𝑡𝑡 )𝛥𝛥𝑡𝑡 = 𝛥𝛥𝐿𝐿 The rate of change of angular momentum is then
𝛥𝛥𝐹𝐹
equal to the net torque. (𝛴𝛴𝜏𝜏𝑑𝑑𝑥𝑥𝑡𝑡 ) = The torque exerted by each
𝛥𝛥𝑡𝑡
force is equal to the magnitude of the force multiplied by the distance
between the application of the force and the pivot point 𝛴𝛴𝜏𝜏 = 𝐹𝐹𝐹𝐹
The largest torque is given by the 60N force as it is three times
further from the pivot as the 150N force.
(D) Incorrect. The change in angular momentum is equal to the net
torque exerted on the object times the time the torque is exerted.
(𝛴𝛴𝜏𝜏𝑑𝑑𝑥𝑥𝑡𝑡 )𝛥𝛥𝑡𝑡 = 𝛥𝛥𝐿𝐿 The rate of change of angular momentum is then
𝛥𝛥𝐹𝐹
equal to the net torque. (𝛴𝛴𝜏𝜏𝑑𝑑𝑥𝑥𝑡𝑡 ) = The torque exerted by each
𝛥𝛥𝑡𝑡
force is equal to the magnitude of the force multiplied by the distance
between the application of the force and the pivot point 𝛴𝛴𝜏𝜏 = 𝐹𝐹𝐹𝐹
The largest torque is given by the 60N force as it is three times
further from the pivot as the 150N force.
Question 28

Science Practice Learning Objective Topic

2.2 3.F.2.1 7.3


4.D.3.1
(A) Incorrect. The 2 N force exerts a torque on the wheel, which
accelerates the wheel. The angular impulse delivered to the wheel
and the final angular speed are related:
𝛥𝛥𝐿𝐿 = 𝜏𝜏net 𝛥𝛥𝑡𝑡
𝐼𝐼𝛥𝛥𝐼𝐼 = 𝑟𝑟𝐹𝐹𝛥𝛥𝑡𝑡
𝐼𝐼(𝐼𝐼 − 0) = 𝑟𝑟𝐹𝐹𝛥𝛥𝑡𝑡
𝐼𝐼𝐼𝐼 = 𝑟𝑟𝐹𝐹𝛥𝛥𝑡𝑡
However, without knowing the rotational inertia 𝐼𝐼 of the wheel, the
final angular speed of the wheel cannot be determined.
(B) Incorrect. The 2 N force exerts a torque on the wheel, which
accelerates the wheel. The angular impulse delivered to the wheel
and the final angular speed are related:
𝛥𝛥𝐿𝐿 = 𝜏𝜏net 𝛥𝛥𝑡𝑡
𝐼𝐼𝛥𝛥𝐼𝐼 = 𝑟𝑟𝐹𝐹𝛥𝛥𝑡𝑡
𝐼𝐼(𝐼𝐼 − 0) = 𝑟𝑟𝐹𝐹𝛥𝛥𝑡𝑡
𝐼𝐼𝐼𝐼 = 𝑟𝑟𝐹𝐹𝛥𝛥𝑡𝑡
However, without knowing the rotational inertia 𝐼𝐼 of the wheel, the
final angular speed of the wheel cannot be determined.
(C) Incorrect. The 2 N force exerts a torque on the wheel, which
accelerates the wheel. The angular impulse delivered to the wheel
and the final angular speed are related:
𝛥𝛥𝐿𝐿 = 𝜏𝜏net 𝛥𝛥𝑡𝑡
𝐼𝐼𝛥𝛥𝐼𝐼 = 𝑟𝑟𝐹𝐹𝛥𝛥𝑡𝑡
𝐼𝐼(𝐼𝐼 − 0) = 𝑟𝑟𝐹𝐹𝛥𝛥𝑡𝑡
𝐼𝐼𝐼𝐼 = 𝑟𝑟𝐹𝐹𝛥𝛥𝑡𝑡
However, without knowing the rotational inertia 𝐼𝐼 of the wheel, the
final angular speed of the wheel cannot be determined.
(D) Correct. The 2 N force exerts a torque on the wheel, which
accelerates the wheel. The angular impulse delivered to the wheel
and the final angular speed are related:
𝛥𝛥𝐿𝐿 = 𝜏𝜏net 𝛥𝛥𝑡𝑡
𝐼𝐼𝛥𝛥𝐼𝐼 = 𝑟𝑟𝐹𝐹𝛥𝛥𝑡𝑡
𝐼𝐼(𝐼𝐼 − 0) = 𝑟𝑟𝐹𝐹𝛥𝛥𝑡𝑡
𝐼𝐼𝐼𝐼 = 𝑟𝑟𝐹𝐹𝛥𝛥𝑡𝑡
However, without knowing the rotational inertia 𝐼𝐼 of the wheel, the
final angular speed of the wheel cannot be determined.
Question 29

Science Practice Learning Objective Topic

6.4 3.F.2.1 7.2


(A) Correct. The force exerted on the rod at the pivot, does not exert a
torque, and so does not change the angular velocity of the rod. The
two forces exerted on either end are equal, but the force exerted on
the right end delivers a larger torque. The net torque on the rod will
cause a clockwise rotation, which is opposite to the initial rotation
direction. The angular speed of the rod must be decreasing
immediately after the forces are exerted. While the rotation of the
rod will eventually stop and change direction, immediately after the
forces are exerted, the rod is rotating in the initial counterclockwise
direction.
(B) Incorrect. The force exerted on the rod at the pivot, does not exert a
torque, and so does not change the angular velocity of the rod. The
two forces exerted on either end are equal, but the force exerted on
the right end delivers a larger torque. The net torque on the rod will
cause a clockwise rotation, which is opposite to the initial rotation
direction. The angular speed of the rod must be decreasing
immediately after the forces are exerted. While the rotation of the
rod will eventually stop and change direction, immediately after the
forces are exerted, the rod is rotating in the initial counterclockwise
direction.
(C) Incorrect. The force exerted on the rod at the pivot, does not exert a
torque, and so does not change the angular velocity of the rod. The
two forces exerted on either end are equal, but the force exerted on
the right end delivers a larger torque. The net torque on the rod will
cause a clockwise rotation, which is opposite to the initial rotation
direction. The angular speed of the rod must be decreasing
immediately after the forces are exerted. While the rotation of the
rod will eventually stop and change direction, immediately after the
forces are exerted, the rod is rotating in the initial counterclockwise
direction.
(D) Incorrect. The force exerted on the rod at the pivot, does not exert a
torque, and so does not change the angular velocity of the rod. The
two forces exerted on either end are equal, but the force exerted on
the right end delivers a larger torque. The net torque on the rod will
cause a clockwise rotation, which is opposite to the initial rotation
direction. The angular speed of the rod must be decreasing
immediately after the forces are exerted. While the rotation of the
rod will eventually stop and change direction, immediately after the
forces are exerted, the rod is rotating in the initial counterclockwise
direction.
Question 30

Science Practice Learning Objective Topic

7.2 3.B.3.1 6.1


(A) Incorrect. Increasing the mass of the pendulum bob does not affect
the period of oscillation of the pendulum.
(B) Incorrect. Increasing the angle through which the pendulum swings
by a small amount does not affect the period of oscillation of the
pendulum.
(C) Correct. The period of oscillation of the mass on a spring and a
𝑏𝑏 ℓ
simple pendulum are 𝑇𝑇𝑑𝑑 = 2𝜋𝜋� and 𝑇𝑇𝑡𝑡 = 2𝜋𝜋� , respectively. The
𝑘𝑘 𝑔𝑔
period of a pendulum is independent of the mass of the pendulum
bob or the angle through which the pendulum swings (for small
angle approximations). Decreasing the mass of the block will
decrease the period which could make it equal to the period of the
pendulum.
(D) Incorrect. Shortening the length of the pendulum would decrease the
period of the pendulum. Since the block-spring system initially has a
larger period, shortening the length of the string cannot help make
the two periods equal.

Question 31

Science Practice Learning Objective Topic

6.4 4.A.2.1 1.2


(A) Incorrect. The slope of a line tangent to the graph of position as a
function of time is equal to the velocity of the object. The time at
which the slope of the lines tangent to their respective position vs.
time graphs are equal is at time 𝑡𝑡𝐶𝐶 .
(B) Incorrect. The slope of a line tangent to the graph of position as a
function of time is equal to the velocity of the object. The time at
which the slope of the lines tangent to their respective position vs.
time graphs are equal is at time 𝑡𝑡𝐶𝐶 .
(C) Correct. The slope of a line tangent to the graph of position as a
function of time is equal to the velocity of the object. The time at
which the slope of the lines tangent to their respective position vs.
time graphs are equal is at time 𝑡𝑡𝐶𝐶 .
(D) Incorrect. The slope of a line tangent to the graph of position as a
function of time is equal to the velocity of the object. The time at
which the slope of the lines tangent to their respective position vs.
time graphs are equal is at time 𝑡𝑡𝐶𝐶 .
Question 32

Science Practice Learning Objective Topic

6.4 4.A.2.2 1.2


(A) Correct. The graph shows the position of each object as a function of
time. Since object 1 is at the same position at 𝑡𝑡𝑓𝑓 , as object 2 was in at
time 𝑡𝑡𝐴𝐴 . The two objects have the same displacement. However,
object 1 travels a longer distance as it goes backwards and forwards
during the time between 𝑡𝑡𝐴𝐴 and 𝑡𝑡𝑓𝑓
(B) Incorrect. The graph shows the position of each object as a function
of time. Since object 1 is at the same position at 𝑡𝑡𝑓𝑓 , as object 2 was in
at time 𝑡𝑡𝐴𝐴 . The two objects have the same displacement. However,
object 1 travels a longer distance as it goes backwards and forwards
during the time between 𝑡𝑡𝐴𝐴 and 𝑡𝑡𝑓𝑓
(C) Incorrect. The graph shows the position of each object as a function
of time. Since object 1 is at the same position at 𝑡𝑡𝑓𝑓 , as object 2 was in
at time 𝑡𝑡𝐴𝐴 . The two objects have the same displacement. However,
object 1 travels a longer distance as it goes backwards and forwards
during the time between 𝑡𝑡𝐴𝐴 and 𝑡𝑡𝑓𝑓
(D) Incorrect. The graph shows the position of each object as a function
of time. Since object 1 is at the same position at 𝑡𝑡𝑓𝑓 , as object 2 was in
at time 𝑡𝑡𝐴𝐴 . The two objects have the same displacement. However,
object 1 travels a longer distance as it goes backwards and forwards
during the time between 𝑡𝑡𝐴𝐴 and 𝑡𝑡𝑓𝑓
Question 33

Science Practice Learning Objective Topic

1.5 3.A.1.1 1.1


3.A.3.1
(A) Incorrect. The acceleration of the ball is proportional to the net force
on the ball, both of which are vector quantities. At time 𝑡𝑡, the ball is
moving up and to the right. The force of air resistance that opposes
the ball’s motion must act down and to the left. The two forces
exerted on the ball are then the force of gravity, which is directed
straight down, and the force of the air which is directed down and to
the left. The net force is the vector sum of these two forces, which is
directed down and left and will have a magnitude greater than the
weight of the ball. The acceleration of the ball is then down and to
the left and is greater than the acceleration due to gravity 𝑔𝑔.
(B) Correct. The acceleration of the ball is proportional to the net force
on the ball, both of which are vector quantities. At time 𝑡𝑡, the ball is
moving up and to the right. The force of air resistance that opposes
the ball’s motion must act down and to the left. The two forces
exerted on the ball are then the force of gravity, which is directed
straight down, and the force of the air which is directed down and to
the left. The net force is the vector sum of these two forces, which is
directed down and left and will have a magnitude greater than the
weight of the ball. The acceleration of the ball is then down and to
the left and is greater than the acceleration due to gravity 𝑔𝑔.
(C) Incorrect. The acceleration of the ball is proportional to the net force
on the ball, both of which are vector quantities. At time 𝑡𝑡, the ball is
moving up and to the right. The force of air resistance that opposes
the ball’s motion must act down and to the left. The two forces
exerted on the ball are then the force of gravity, which is directed
straight down, and the force of the air which is directed down and to
the left. The net force is the vector sum of these two forces, which is
directed down and left and will have a magnitude greater than the
weight of the ball. The acceleration of the ball is then down and to
the left and is greater than the acceleration due to gravity 𝑔𝑔.
(D) Incorrect. The acceleration of the ball is proportional to the net force
on the ball, both of which are vector quantities. At time 𝑡𝑡, the ball is
moving up and to the right. The force of air resistance that opposes
the ball’s motion must act down and to the left. The two forces
exerted on the ball are then the force of gravity, which is directed
straight down, and the force of the air which is directed down and to
the left. The net force is the vector sum of these two forces, which is
directed down and left and will have a magnitude greater than the
weight of the ball. The acceleration of the ball is then down and to
the left and is greater than the acceleration due to gravity 𝑔𝑔.
Question 34

Science Practice Learning Objective Topic

2.2 4.A.1.1 5.4


4.A.2.1
4.A.3.2
5.D.2.5
5.D.3.1
(A) Incorrect. The initial speed of the two-skater system can be
𝛥𝛥𝑥𝑥 4ticks
calculated from the figure as 𝑣𝑣0,together = = = 2.0 ticks�s
𝛥𝛥𝑡𝑡 2s
The initial momentum of the two-skater system is then
𝑝𝑝0 = 𝜇𝜇𝑡𝑡𝑡𝑡𝑡𝑡 𝑣𝑣0,together = (2𝜇𝜇 + 𝜇𝜇)𝑣𝑣0,together
𝑝𝑝0 = 3𝜇𝜇 �2.0 ticks�s�
There are no external forces on the two-skater system, so after they
push off each other, the total momentum of the two-skater system
does not change. At 𝑡𝑡 = 4s, the larger skater has a momentum of
(2𝜇𝜇) �1 ticks�s�, which means that the smaller skater must have a
momentum of (4 m · ticks�s). Because the mass of the smaller skater
is half that of the larger skater, the speed of the smaller skater must
be four times the speed of the larger skater and would be at point D
at 𝑡𝑡 = 4s.
(B) Incorrect. The initial speed of the two-skater system can be
𝛥𝛥𝑥𝑥 4ticks
calculated from the figure as 𝑣𝑣0,together = = = 2.0 ticks�s
𝛥𝛥𝑡𝑡 2s
The initial momentum of the two-skater system is then
𝑝𝑝0 = 𝜇𝜇𝑡𝑡𝑡𝑡𝑡𝑡 𝑣𝑣0,together = (2𝜇𝜇 + 𝜇𝜇)𝑣𝑣0,together
𝑝𝑝0 = 3𝜇𝜇 �2.0 ticks�s�
There are no external forces on the two-skater system, so after they
push off each other, the total momentum of the two-skater system
does not change. At 𝑡𝑡 = 4s, the larger skater has a momentum of
(2𝜇𝜇) �1 ticks�s�, which means that the smaller skater must have a
momentum of (4 m · ticks�s). Because the mass of the smaller skater
is half that of the larger skater, the speed of the smaller skater must
be four times the speed of the larger skater and would be at point D
at 𝑡𝑡 = 4s.
Question 34 (continued)
(C) Incorrect. The initial speed of the two-skater system can be
𝛥𝛥𝑥𝑥 4ticks
calculated from the figure as 𝑣𝑣0,together = = = 2.0 ticks�s
𝛥𝛥𝑡𝑡 2s
The initial momentum of the two-skater system is then
𝑝𝑝0 = 𝜇𝜇𝑡𝑡𝑡𝑡𝑡𝑡 𝑣𝑣0,together = (2𝜇𝜇 + 𝜇𝜇)𝑣𝑣0,together
𝑝𝑝0 = 3𝜇𝜇 �2.0 ticks�s�
There are no external forces on the two-skater system, so after they
push off each other, the total momentum of the two-skater system
does not change. At 𝑡𝑡 = 4s, the larger skater has a momentum of
(2𝜇𝜇) �1 ticks�s�, which means that the smaller skater must have a
momentum of (4 m · ticks�s). Because the mass of the smaller skater
is half that of the larger skater, the speed of the smaller skater must
be four times the speed of the larger skater and would be at point D
at 𝑡𝑡 = 4s.
(D) Correct. The initial speed of the two-skater system can be calculated
𝛥𝛥𝑥𝑥 4ticks
from the figure as 𝑣𝑣0,together = = = 2.0 ticks�s
𝛥𝛥𝑡𝑡 2s
The initial momentum of the two-skater system is then
𝑝𝑝0 = 𝜇𝜇𝑡𝑡𝑡𝑡𝑡𝑡 𝑣𝑣0,together = (2𝜇𝜇 + 𝜇𝜇)𝑣𝑣0,together
𝑝𝑝0 = 3𝜇𝜇 �2.0 ticks�s�
There are no external forces on the two-skater system, so after they
push off each other, the total momentum of the two-skater system
does not change. At 𝑡𝑡 = 4s, the larger skater has a momentum of
(2𝜇𝜇) �1 ticks�s�, which means that the smaller skater must have a
momentum of (4 m · ticks�s). Because the mass of the smaller skater
is half that of the larger skater, the speed of the smaller skater must
be four times the speed of the larger skater and would be at point D
at 𝑡𝑡 = 4s.
Question 35

Science Practice Learning Objective Topic

6.4 3.F.2.1 7.2


(A) Correct. All the forces have identical magnitudes but are exerted in
different directions. The greatest increase in angular speed will come
from the force that exerts the greatest torque.
𝛥𝛥𝐿𝐿 = 𝜏𝜏net 𝛥𝛥𝑡𝑡
𝐼𝐼𝛥𝛥𝐼𝐼 = 𝜏𝜏net 𝛥𝛥𝑡𝑡 .
𝜏𝜏net = 𝑟𝑟𝐹𝐹sin𝜃𝜃
All three forces are exerted at the same distance from the axis of
rotation and have the same magnitude. Therefore, the greatest torque
comes from force 𝐹𝐹𝐴𝐴 because it is directed at a 90° angle relative to
the radial direction.
(B) Incorrect. All the forces have identical magnitudes but are exerted in
different directions. The greatest increase in angular speed will come
from the force that exerts the greatest torque.
𝛥𝛥𝐿𝐿 = 𝜏𝜏net 𝛥𝛥𝑡𝑡
𝐼𝐼𝛥𝛥𝐼𝐼 = 𝜏𝜏net 𝛥𝛥𝑡𝑡 .
𝜏𝜏net = 𝑟𝑟𝐹𝐹sin𝜃𝜃
All three forces are exerted at the same distance from the axis of
rotation and have the same magnitude. Therefore, the greatest torque
comes from force 𝐹𝐹𝐴𝐴 because it is directed at a 90° angle relative to
the radial direction.
(C) Incorrect. All the forces have identical magnitudes but are exerted in
different directions. The greatest increase in angular speed will come
from the force that exerts the greatest torque.
𝛥𝛥𝐿𝐿 = 𝜏𝜏net 𝛥𝛥𝑡𝑡
𝐼𝐼𝛥𝛥𝐼𝐼 = 𝜏𝜏net 𝛥𝛥𝑡𝑡 .
𝜏𝜏net = 𝑟𝑟𝐹𝐹sin𝜃𝜃
All three forces are exerted at the same distance from the axis of
rotation and have the same magnitude. Therefore, the greatest torque
comes from force 𝐹𝐹𝐴𝐴 because it is directed at a 90° angle relative to
the radial direction.
(D) Incorrect. All the forces have identical magnitudes but are exerted in
different directions. The greatest increase in angular speed will come
from the force that exerts the greatest torque.
𝛥𝛥𝐿𝐿 = 𝜏𝜏net 𝛥𝛥𝑡𝑡
𝐼𝐼𝛥𝛥𝐼𝐼 = 𝜏𝜏net 𝛥𝛥𝑡𝑡 .
𝜏𝜏net = 𝑟𝑟𝐹𝐹sin𝜃𝜃
All three forces are exerted at the same distance from the axis of
rotation and have the same magnitude. Therefore, the greatest torque
comes from force 𝐹𝐹𝐴𝐴 because it is directed at a 90° angle relative to
the radial direction.
Question 36

Science Practice Learning Objective Topic

6.4 5.B.5.3 4.3


(A) Incorrect. The only force doing work on the object is the force given
in the graphs. Using the force vs. position graphs shown, the net
work done on the object is equal to the area bounded by the force
curve and the horizontal axis. The work done on the object in trial 2
is therefore larger than the work done on the object in trial 1.
The work done on the object by a variable force can be found by
𝑊𝑊 = 𝐹𝐹avg 𝐹𝐹cos𝜃𝜃 This means that because 𝑊𝑊1 < 𝑊𝑊2 and 𝐹𝐹1 = 𝐹𝐹2
the average force exerted in trial 1 must be less than the average
force exerted in trial 2.
(B) Incorrect. The only force doing work on the object is the force given
in the graphs. Using the force vs. position graphs shown, the net
work done on the object is equal to the area bounded by the force
curve and the horizontal axis. The work done on the object in trial 2
is therefore larger than the work done on the object in trial 1.
The work done on the object by a variable force can be found by
𝑊𝑊 = 𝐹𝐹avg 𝐹𝐹cos𝜃𝜃 This means that because 𝑊𝑊1 < 𝑊𝑊2 and 𝐹𝐹1 = 𝐹𝐹2
the average force exerted in trial 1 must be less than the average
force exerted in trial 2.
(C) Correct. The only force doing work on the object is the force given
in the graphs. Using the force vs. position graphs shown, the net
work done on the object is equal to the area bounded by the force
curve and the horizontal axis. The work done on the object in trial 2
is therefore larger than the work done on the object in trial 1.
The work done on the object by a variable force can be found by
𝑊𝑊 = 𝐹𝐹avg 𝐹𝐹cos𝜃𝜃 This means that because 𝑊𝑊1 < 𝑊𝑊2 and 𝐹𝐹1 = 𝐹𝐹2
the average force exerted in trial 1 must be less than the average
force exerted in trial 2.
(D) Incorrect. The only force doing work on the object is the force given
in the graphs. Using the force vs. position graphs shown, the net
work done on the object is equal to the area bounded by the force
curve and the horizontal axis. The work done on the object in trial 2
is therefore larger than the work done on the object in trial 1.
The work done on the object by a variable force can be found by
𝑊𝑊 = 𝐹𝐹avg 𝐹𝐹cos𝜃𝜃 This means that because 𝑊𝑊1 < 𝑊𝑊2 and 𝐹𝐹1 = 𝐹𝐹2
the average force exerted in trial 1 must be less than the average
force exerted in trial 2.
Question 37

Science Practice Learning Objective Topic

1.4 4.A.1.1 1.2


(A) Correct. The cars are identical, and so have identical masses. The
velocity of each car can be determined from the slope of the position
m m
30 -10 𝑏𝑏
vs. time graph. The velocity of car 1 is equal to � s s
� = +20 .
1.0 s 𝑑𝑑
m m
-10 -10 𝑏𝑏
The velocity of car 2 is � s s
� = −20 . The cars are identical
1.0 s 𝑑𝑑
and are moving at the same speeds and in opposite directions, so the
speed of the center of mass is equal to zero.
(B) Incorrect. The cars are identical, and so have identical masses. The
velocity of each car can be determined from the slope of the position
m m
30 -10 𝑏𝑏
vs. time graph. The velocity of car 1 is equal to � s s
� = +20 .
1.0 s 𝑑𝑑
m m
-10 -10 𝑏𝑏
The velocity of car 2 is � s s
� = −20 . The cars are identical
1.0 s 𝑑𝑑
and are moving at the same speeds and in opposite directions, so the
speed of the center of mass is equal to zero.
(C) Incorrect. The cars are identical, and so have identical masses. The
velocity of each car can be determined from the slope of the position
m m
30 -10 𝑏𝑏
vs. time graph. The velocity of car 1 is � s s
� = +20 . The
1.0 s 𝑑𝑑
m m
-10 -10 𝑏𝑏
velocity of car 2 is � s s
� = −20 . The cars are identical and are
1.0 s 𝑑𝑑
moving at the same speeds and in opposite directions, so the speed of
the center of mass is equal to zero.
(D) Incorrect. The cars are identical, and so have identical masses. The
velocity of each car can be determined from the slope of the position
m m
30 -10 𝑏𝑏
vs. time graph. The velocity of car 1 is equal to � s s
� = +20 .
1.0 s 𝑑𝑑
m m
-10 -10 𝑏𝑏
The velocity of car 2 is � s s
� = −20 . The cars are identical
1.0 s 𝑑𝑑
and are moving at the same speeds and in opposite directions, so the
speed of the center of mass is equal to zero.
Question 38

Science Practice Learning Objective Topic

7.2 5.E.1.1 7.4


(A) Correct. The only force acting on the spheres-rod system while it is
in the air is the force of gravity which exerts no torque on the system
because it acts at the center of mass. There is no net torque on the
system, therefore the angular momentum remains constant.
(B) Incorrect. The only force acting on the spheres-rod system while it is
in the air is the force of gravity which exerts no torque on the system
because it acts at the center of mass. There is no net torque on the
system, therefore the angular momentum remains constant.
(C) Incorrect. The only force acting on the spheres-rod system while it is
in the air is the force of gravity which exerts no torque on the system
because it acts at the center of mass. There is no net torque on the
system, therefore the angular momentum remains constant.
(D) Incorrect. The only force acting on the spheres-rod system while it is
in the air is the force of gravity which exerts no torque on the system
because it acts at the center of mass. There is no net torque on the
system, therefore the angular momentum remains constant.
Question 39

Science Practice Learning Objective Topic

2.2 3.E.1.4 4.2


(A) Incorrect. The area under a force graph as a function of displacement
is equal to the work done on the box which is equal to:
1
𝑊𝑊 = 𝐹𝐹𝐹𝐹 = (12N)(6m) = 36Nm.
2
There are no other forces doing work on the box, and the box was
initially at rest, so the work is calculated above is equal to the final
kinetic energy of the box, and the final the speed of the object after
being pushed for can be determined.
1 1
36Nm = 𝜇𝜇𝑣𝑣 2 = (2kg)(v2 )
2 2
m .
𝑣𝑣 = 6
s
The change in momentum of the box is equal to
m m kgm
𝛥𝛥𝑝𝑝 = 𝑝𝑝𝑓𝑓 − 𝑝𝑝𝑑𝑑 = (2kg)(6 ) − (2kg)(0 ) = 12
s s s
(B) Incorrect. The area under a force graph as a function of displacement
is equal to the work done on the box which is equal to:
1
𝑊𝑊 = 𝐹𝐹𝐹𝐹 = (12N)(6m) = 36Nm.
2
There are no other forces doing work on the box, and the box was
initially at rest, so the work is calculated above is equal to the final
kinetic energy of the box, and the final the speed of the object after
being pushed for can be determined.
1 1
36Nm = 𝜇𝜇𝑣𝑣 2 = (2kg)(v2 )
2 2
m .
𝑣𝑣 = 6
s
The change in momentum of the box is equal to
m m kgm
𝛥𝛥𝑝𝑝 = 𝑝𝑝𝑓𝑓 − 𝑝𝑝𝑑𝑑 = (2kg)(6 ) − (2kg)(0 ) = 12
s s s
Question 39 (continued)

(C) Correct. The area under a force graph as a function of displacement


is equal to the work done on the box which is equal to:
1
𝑊𝑊 = 𝐹𝐹𝐹𝐹 = (12N)(6m) = 36Nm.
2
There are no other forces doing work on the box, and the box was
initially at rest, so the work is calculated above is equal to the final
kinetic energy of the box, and the final the speed of the object after
being pushed for can be determined.
1 1
36Nm = 𝜇𝜇𝑣𝑣 2 = (2kg)(v2 )
2 2
m .
𝑣𝑣 = 6
s
The change in momentum of the box is equal to
m m kgm
𝛥𝛥𝑝𝑝 = 𝑝𝑝𝑓𝑓 − 𝑝𝑝𝑑𝑑 = (2kg)(6 ) − (2kg)(0 ) = 12
s s s
(D) Incorrect. The area under a force graph as a function of displacement
is equal to the work done on the box which is equal to:
1
𝑊𝑊 = 𝐹𝐹𝐹𝐹 = (12N)(6m) = 36Nm.
2
There are no other forces doing work on the box, and the box was
initially at rest, so the work is calculated above is equal to the final
kinetic energy of the box, and the final the speed of the object after
being pushed for can be determined.
1 1
36Nm = 𝜇𝜇𝑣𝑣 2 = (2kg)(v2 )
2 2
m .
𝑣𝑣 = 6
s
The change in momentum of the box is equal to
m m kgm
𝛥𝛥𝑝𝑝 = 𝑝𝑝𝑓𝑓 − 𝑝𝑝𝑑𝑑 = (2kg)(6 ) − (2kg)(0 ) = 12
s s s
Question 40

Science Practice Learning Objective Topic

6.4 4.A.2.1 1.2


(A) Correct. Between 𝑡𝑡1 and 𝑡𝑡2 , there is a net force on block 2 exerted
from the spring, because block 1 collides with and compresses the
spring. This causes a rightward acceleration that increases and
returns to zero as the spring compresses and returns to its original
length during the time 𝑡𝑡1 to 𝑡𝑡2 . There are no external forces acting
on the blocks-spring system, so the center of mass of the two-block
system during the time 𝑡𝑡1 to 𝑡𝑡2 .
(B) Incorrect. Between 𝑡𝑡1 and 𝑡𝑡2 , there is a net force on block 2 exerted
from the spring, because block 1 collides with and compresses the
spring. This causes a rightward acceleration that increases and
returns to zero as the spring compresses and returns to its original
length during the time 𝑡𝑡1 to 𝑡𝑡2 . There are no external forces acting
on the blocks-spring system, so the center of mass of the two-block
system during the time 𝑡𝑡1 to 𝑡𝑡2 .
(C) Incorrect. Between 𝑡𝑡1 and 𝑡𝑡2 , there is a net force on block 2 exerted
from the spring, because block 1 collides with and compresses the
spring. This causes a rightward acceleration that increases and
returns to zero as the spring compresses and returns to its original
length during the time 𝑡𝑡1 to 𝑡𝑡2 . There are no external forces acting
on the blocks-spring system, so the center of mass of the two-block
system during the time 𝑡𝑡1 to 𝑡𝑡2 .
(D) Incorrect. Between 𝑡𝑡1 and 𝑡𝑡2 , there is a net force on block 2 exerted
from the spring, because block 1 collides with and compresses the
spring. This causes a rightward acceleration that increases and
returns to zero as the spring compresses and returns to its original
length during the time 𝑡𝑡1 to 𝑡𝑡2 . There are no external forces acting
on the blocks-spring system, so the center of mass of the two-block
system during the time 𝑡𝑡1 to 𝑡𝑡2 .
Question 41

Science Practice Learning Objective Topic

4.2 3.D.2.4 5.1


(A) Correct. The force sensor will record force as a function of time
exerted on block 2 by block 1. The area under this graph is equal to
the change in momentum of block 2. Because block 2 is initially at
rest, no additional measurements are needed to determine the
momentum of block 2 after the collision.
(B) Incorrect. The force sensor will record force as a function of time
exerted on block 2 by block 1. The area under this graph is equal to
the change in momentum of block 2. Because block 2 is initially at
rest, no additional measurements are needed to determine the
momentum of block 2 after the collision.
(C) Incorrect. The force sensor will record force as a function of time
exerted on block 2 by block 1. The area under this graph is equal to
the change in momentum of block 2. Because block 2 is initially at
rest, no additional measurements are needed to determine the
momentum of block 2 after the collision.
(D) Incorrect. The force sensor will record force as a function of time
exerted on block 2 by block 1. The area under this graph is equal to
the change in momentum of block 2. Because block 2 is initially at
rest, no additional measurements are needed to determine the
momentum of block 2 after the collision.
Question 42

Science Practice Learning Objective Topic

1.4 3.F.1.5 7.2


(A) Incorrect. The meterstick will rotate about an axis through its center
of mass, which is halfway between locations B and C. The two forces
exerted at near the ends of the meterstick are equidistant from the
center of mass so exert torques that are equal in magnitude but in
opposite directions, resulting in a combined torque from these two
forces of zero. The force that is exerted at point C creates a torque
that is counterclockwise. For the meterstick to be in equilibrium, an
additional force of magnitude 𝐹𝐹0 directed upward should be exerted
at point B. This would cause the net force exerted on the meterstick
to be zero, and create a torque equal in magnitude but in the
opposite direction to the torque created by the force exerted on the
meterstick at point C. The meterstick will then be in translational
and rotational equilibrium.
(B) Correct. The meterstick will rotate about an axis through its center
of mass, which is halfway between locations B and C. The two forces
exerted at near the ends of the meterstick are equidistant from the
center of mass so exert torques that are equal in magnitude but in
opposite directions, resulting in a combined torque from these two
forces of zero. The force that is exerted at point C creates a torque
that is counterclockwise. For the meterstick to be in equilibrium, an
additional force of magnitude 𝐹𝐹0 directed upward should be exerted
at point B. This would cause the net force exerted on the meterstick
to be zero, and create a torque equal in magnitude but in the
opposite direction to the torque created by the force exerted on the
meterstick at point C. The meterstick will then be in translational
and rotational equilibrium.
Question 42 (continued)
(C) Incorrect. The meterstick will rotate about an axis through its center
of mass, which is halfway between locations B and C. The two forces
exerted at near the ends of the meterstick are equidistant from the
center of mass so exert torques that are equal in magnitude but in
opposite directions, resulting in a combined torque from these two
forces of zero. The force that is exerted at point C creates a torque
that is counterclockwise. For the meterstick to be in equilibrium, an
additional force of magnitude 𝐹𝐹0 directed upward should be exerted
at point B. This would cause the net force exerted on the meterstick
to be zero, and create a torque equal in magnitude but in the
opposite direction to the torque created by the force exerted on the
meterstick at point C. The meterstick will then be in translational
and rotational equilibrium.
(D) Incorrect. The meterstick will rotate about an axis through its center
of mass, which is halfway between locations B and C. The two forces
exerted at near the ends of the meterstick are equidistant from the
center of mass so exert torques that are equal in magnitude but in
opposite directions, resulting in a combined torque from these two
forces of zero. The force that is exerted at point C creates a torque
that is counterclockwise. For the meterstick to be in equilibrium, an
additional force of magnitude 𝐹𝐹0 directed upward should be exerted
at point B. This would cause the net force exerted on the meterstick
to be zero, and create a torque equal in magnitude but in the
opposite direction to the torque created by the force exerted on the
meterstick at point C. The meterstick will then be in translational
and rotational equilibrium.
Question 43

Science Practice Learning Objective Topic

1.4 4.B.1.1 5.2


(A) Correct. The initial momentum of the system is equal to
m kg·m
(3kg) �2 � = 6 . After the compressed spring expands,
s s
the momentum of the system is equal to
m m kg·m
(2kg) �2.5 � + (1kg) �1 � = 6 . There is no change in linear
s s s
momentum of the system because of the spring expanding. This
matches our expectation since the spring exerts equal and opposite
forces on each cart and so it exerts no net force on the two-cart
system.
(B) Incorrect – There are no net external forces on the two-cart system,
so the linear momentum of the system does not change. This is
supported by the data shown in the figure.
(C) Incorrect – There are no net external forces on the two-cart system,
so the linear momentum of the system does not change. This is
supported by the data shown in the figure.
(D) Incorrect – There are no net external forces on the two-cart system,
so the linear momentum of the system does not change. This is
supported by the data shown in the figure.
Question 44

Science Practice Learning Objective Topic

6.4 3.A.3.1 3.8


(A) Incorrect. There are two forces exerted on the cart at point P, the
gravitational force, which is exerted vertically downwards and the
normal force from the track, which at point P is exerted vertically
upwards. At point P the cart is moving along a circular path, and so
is accelerating towards the center of that circular path. The
acceleration of the cart must be directed upwards, and so the net
force must be vertically upward. The net force can only be upwards if
the normal force from the track is larger than the weight.
(B) Incorrect. There are two forces exerted on the cart at point P, the
gravitational force, which is exerted vertically downwards and the
normal force from the track, which at point P is exerted vertically
upwards. At point P the cart is moving along a circular path, and so
is accelerating towards the center of that circular path. The
acceleration of the cart must be directed upwards, and so the net
force must be vertically upward. The net force can only be upwards if
the normal force from the track is larger than the weight.
(C) Incorrect. There are two forces exerted on the cart at point P, the
gravitational force, which is exerted vertically downwards and the
normal force from the track, which at point P is exerted vertically
upwards. At point P the cart is moving along a circular path, and so
is accelerating towards the center of that circular path. The
acceleration of the cart must be directed upwards, and so the net
force must be vertically upward. The net force can only be upwards if
the normal force from the track is larger than the weight.
(D) Correct. There are two forces exerted on the cart at point P, the
gravitational force, which is exerted vertically downwards and the
normal force from the track, which at point P is exerted vertically
upwards. At point P the cart is moving along a circular path, and so
is accelerating towards the center of that circular path. The
acceleration of the cart must be directed upwards, and so the net
force must be vertically upward. The net force can only be upwards if
the normal force from the track is larger than the weight.
Question 45

Science Practice Learning Objective Topic

6.4 5.E.1.1 7.4


(A) Correct. The change in angular momentum of the meterstick is
equal to the net torque exerted on the meterstick times the time for
which that torque is exerted. In this case, there are 5 forces exerted
on the rod. The normal force upward from the pivot acts at the pivot
and so exerts no torque. The force applied from the 1.0 kg object
which is equal to 10 N, gives a torque equal to
𝜏𝜏 = 𝐹𝐹𝐹𝐹sin𝜃𝜃 = (10 N)(0.25m)sin90° = 2.5Nm.
This torque is exerted so that it would cause a counterclockwise
rotation. The force of gravity exerted on the rod itself is equal to
the weight of the rod, and causes a torque equal to
𝜏𝜏 = 𝐹𝐹𝐹𝐹sin𝜃𝜃 = (5 N)(0.25m)sin90° = 1.25Nm, and will cause a
clockwise rotation. The 0.5 kg object also exerts a torque on the rod
equal to 𝜏𝜏 = 𝐹𝐹𝐹𝐹sin𝜃𝜃 = (5 N)(0.25m)sin90° = 1.25Nm, also in a
clockwise direction. The net torque then on the rod is equal to zero.
The rod will not rotate when released, and its angular momentum
will not change.
(B) Incorrect. The change in angular momentum of the meterstick is
equal to the net torque exerted on the meterstick times the time for
which that torque is exerted. In this case, there are 5 forces exerted
on the rod. The normal force upward from the pivot acts at the pivot
and so exerts no torque. The force applied from the 1.0 kg object
which is equal to 10 N, gives a torque equal to
𝜏𝜏 = 𝐹𝐹𝐹𝐹sin𝜃𝜃 = (10 N)(0.25m)sin90° = 2.5Nm.
This torque is exerted so that it would cause a counterclockwise
rotation. The force of gravity exerted on the rod itself is equal to
the weight of the rod, and causes a torque equal to
𝜏𝜏 = 𝐹𝐹𝐹𝐹sin𝜃𝜃 = (5 N)(0.25m)sin90° = 1.25Nm, and will cause a
clockwise rotation. The 0.5 kg object also exerts a torque on the rod
equal to 𝜏𝜏 = 𝐹𝐹𝐹𝐹sin𝜃𝜃 = (5 N)(0.25m)sin90° = 1.25Nm, also in a
clockwise direction. The net torque then on the rod is equal to zero.
The rod will not rotate when released, and its angular momentum
will not change.
Question 45 (continued)
(C) Incorrect. The change in angular momentum of the meterstick is
equal to the net torque exerted on the meterstick times the time for
which that torque is exerted. In this case, there are 5 forces exerted
on the rod. The normal force upward from the pivot acts at the pivot
and so exerts no torque. The force applied from the 1.0 kg object
which is equal to 10 N, gives a torque equal to
𝜏𝜏 = 𝐹𝐹𝐹𝐹sin𝜃𝜃 = (10 N)(0.25m)sin90° = 2.5Nm.
This torque is exerted so that it would cause a counterclockwise
rotation. The force of gravity exerted on the rod itself is equal to
the weight of the rod, and causes a torque equal to
𝜏𝜏 = 𝐹𝐹𝐹𝐹sin𝜃𝜃 = (5 N)(0.25m)sin90° = 1.25Nm, and will cause a
clockwise rotation. The 0.5 kg object also exerts a torque on the rod
equal to 𝜏𝜏 = 𝐹𝐹𝐹𝐹sin𝜃𝜃 = (5 N)(0.25m)sin90° = 1.25Nm, also in a
clockwise direction. The net torque then on the rod is equal to zero.
The rod will not rotate when released, and its angular momentum
will not change.
(D) Incorrect. The change in angular momentum of the meterstick is
equal to the net torque exerted on the meterstick times the time for
which that torque is exerted. In this case, there are 5 forces exerted
on the rod. The normal force upward from the pivot acts at the pivot
and so exerts no torque. The force applied from the 1.0 kg object
which is equal to 10 N, gives a torque equal to
𝜏𝜏 = 𝐹𝐹𝐹𝐹sin𝜃𝜃 = (10 N)(0.25m)sin90° = 2.5Nm.
This torque is exerted so that it would cause a counterclockwise
rotation. The force of gravity exerted on the rod itself is equal to
the weight of the rod, and causes a torque equal to
𝜏𝜏 = 𝐹𝐹𝐹𝐹sin𝜃𝜃 = (5 N)(0.25m)sin90° = 1.25Nm, and will cause a
clockwise rotation. The 0.5 kg object also exerts a torque on the rod
equal to 𝜏𝜏 = 𝐹𝐹𝐹𝐹sin𝜃𝜃 = (5 N)(0.25m)sin90° = 1.25Nm, also in a
clockwise direction. The net torque then on the rod is equal to zero.
The rod will not rotate when released, and its angular momentum
will not change.
Question 131

Science Practice Learning Objective Topic

1.4 3.A.4.3 2.6


3.B.2.1
(A) Correct. This answer correctly relates the horizontal forces exerted
on and the resulting horizontal acceleration of the skier. The tension
force exerted by the rope on the skier minus the drag force exerted
by the water equals the horizontal acceleration of the skier.
(B) Incorrect. Because the skier is not accelerating in the vertical
direction, the net force in the vertical direction should be equal to
zero.
(C) Correct. This answer correctly relates the vertical forces exerted on
and the resulting vertical acceleration of the skier. The lift force from
the water is exerted upwards, while the force of gravity on the skier is
exerted downwards. Since the skier is not accelerating in the vertical
direction, the difference of these two forces is equal to zero.
(D) Incorrect. Since the skier is accelerating in the horizontal direction,
the net force, 𝑇𝑇 − 𝐹𝐹𝑑𝑑 , does not equal zero.

Question 132

Science Practice Learning Objective Topic

5.1 3.D.2.2 5.1


3.D.2.3
(A) Incorrect. Both cars begin to slow as soon as the force is applied,
which in both cases, is before 10.05s.
(B) Correct. The impulse exerted by the wall is equal to the change in the
momentum of each cart. Acceleration is proportional to the force
exerted on each cart by the wall, so the impulse exerted on each cart
is proportional to the area bounded by the acceleration as a function
of time. Car B accelerates for approximately half the time, with twice
the maximum acceleration and so experiences approximately the
same change in velocity as car A.
(C) Incorrect. The net external force is exerted for a shorter time on car
B.
(D) Correct. The data shown in the graphs indicate that the time interval
over which Car B accelerates (10.0s - 10.1 s) is less than that for car
A (10.3s - 10.07 s).
Question 133

Science Practice Learning Objective Topic

2.2 3.A.1.1 1.1


(A) Correct. The total distance traveled by the truck can be found using
𝐹𝐹 = 𝑣𝑣avg 𝑡𝑡. The average value of the speed as a function of time graph
shown is 18 m⁄s, and the time interval over which the truck is
moving is 10s.
(B) Incorrect. Although the initial speed is 30 m⁄s, the average speed is
not 15 m⁄s, so multiplying 15 m⁄s by the 10s travel time will not
yield the total distance traveled by the truck.
(C) Incorrect. Although the acceleration of the truck can be determined
1
from the slope of the graph, the equation 𝐹𝐹 = 𝑎𝑎𝑡𝑡 2 does not take
2
into account the initial speed of the truck.
(D) Correct. The area under a speed vs. time graph is equal to the
distance traveled by the truck during the time interval shown.

Question 134

Science Practice Learning Objective Topic

7.2 2.B.1.1 3.4


2.B.2.1
(A) Incorrect. The weight of the vehicle is equal to the mass of the vehicle
times the gravitational field strength, which changes as the vehicle
approaches the surface of Mars from far away.
(B) Correct. The weight of the vehicle is equal to the gravitational field
strength times the mass of the vehicle. As the vehicle descends to the
surface of Mars, the gravitational field strength increases, and so the
weight of the vehicle also increases.
(C) Incorrect. While the weight of the vehicle is sometimes equal to the
normal force exerted by the surface of Mars, the normal force can
change, and can be larger than, less than or equal to the weight of the
vehicle depending on the vertical acceleration of the vehicle.
(D) Correct. The weight on the surface of Mars is equal to the
gravitational field strength times the mass of the vehicle. Since the
gravitational field strength on Mars is less than the gravitational field
strength on Earth, the vehicle weighs less on the surface on Mars
than it does on the surface of Earth.
Question 135

Science Practice Learning Objective Topic

6.4 4.C.1.2 4.2


(A) Incorrect. For the system of the block and Earth, the block should be
closer to the earth at time 𝑡𝑡1 than it is at time 𝑡𝑡2 .
(B) Correct. Friction is negligible, so the sum of the gravitational
potential energy and kinetic energy of the block-earth system is
constant. Compared to 𝑡𝑡1 , the position of the block at 𝑡𝑡2 is farther
away from Earth and the speed of the block has decreased. The
energy bar charts provided correctly indicate that the total
mechanical energy is constant, the gravitational potential energy
increases, and the kinetic energy decreases.
(C) Incorrect. Since friction is not negligible, the bar charts for this
situation would not have a constant mechanical energy, and we
would expect that the mechanical energy at time 𝑡𝑡2 should be less
than the mechanical energy at 𝑡𝑡1 .
(D) Correct. The cylinder rolls without slipping, so no energy is
dissipated by friction as thermal energy. The sum of the gravitational
potential energy and kinetic energy of the cylinder-earth system is
constant. Compared to 𝑡𝑡1 , the position of the cylinder at 𝑡𝑡2 is farther
away from Earth and the rotational and translational speeds of the
cylinder have decreased. The energy bar charts provided correctly
indicate that the total mechanical energy is constant, the
gravitational potential energy increases, and the total kinetic energy
decreases.
AP® Physics 1 2022 Scoring Guidelines

Question 1: Short Answer 7 points

(a) Horizontal component: For a non-zero horizontal line 1 point

Vertical component: For a non-vertical line with a negative slope that crosses the 1 point
horizontal axis near t f 2 and reaches t f
Scoring note: Any part of the sketched graph beyond t f is ignored.

Total for part (a) 2 points


(b) (i) For a gravitational force with correct direction and label 1 point
For a tension force with correct direction and label 1 point

Scoring note: A maximum of one point may be earned if there are any extraneous forces.

(ii) For a vector pointing downward representing the gravitational force with no extraneous 1 point
forces

Scoring note: A label is not required, since the use of labels is assessed in part (b)(i).

(iii) For attempting to find the horizontal component of velocity 1 point


Scoring note: See examples below. This point is earned for the wrong use of trig
functions if it is clear it is being used to find the horizontal component, but not awarded if
it’s not clear which component the trig function is referring to, or for stating that the
vertical component is zero.

For a correct numerical answer for horizontal velocity with correct units 1 point

Example response:
0 points: v sin θ = 13 m s (No clear indication of an attempt to find the horizontal
component)
1 point:
= sin θ 13 m s (An attempt to find the horizontal component is implied
vx v=
by the vx term)
1 point: v cos θ (An attempt to find the horizontal component is implied
by the cos θ term)
1 point: v cos θ = 7.5 (Correct numerical answer with no units)
1 point: 7.5 m s (Correct answer with units, but no work shown)
2 points: 15cos 60 = 7.5 m s (Correct answer with units)
2 points: 15cos 60 m s (Correct answer with units)

Total for part (b) 5 points

Total for question 1 7 points

© 2022 College Board


AP® Physics 1 2022 Scoring Guidelines

Question 2: Experimental Design 12 points

(a) For a valid statement relating the speed of cart and dissipation of mechanical energy 1 point

Examples of valid statements:


More damage when going faster
Slower collision means compression and decompression during collision happen
without permanent deformation
Examples of invalid statements:
Higher fraction of K is lost at higher speeds
Because more kinetic energy is lost
Total for part (a) 1 point

(b) For measuring the speed of both carts before and after collision 1 point
Scoring note: It is not necessary to measure the initial speed of the cart that is at rest.

For measuring the mass of the carts (or calculating mass ratio using conservation of 1 point
momentum, which may be stated later)

For equipment and measurements consistent with procedure as described or drawn in 1 point
diagram (e.g., motion detectors in the right places to measure speed of both carts)

For varying the speed (making speed the independent variable) of the first cart in a 1 point
feasible experiment that could be done in a school lab.
Scoring note: “Varying the speed” means at least 2 trials (to test hypothesis), and
“feasible” means that equipment is used appropriately for each measurement.
For attempting to reduce uncertainty, e.g., multiple trials for a given initial speed of cart 1,
or trials involving at least three different initial speeds

© 2022 College Board


AP® Physics 1 2022 Scoring Guidelines

For attempting to reduce uncertainty, e.g., multiple trials for a given initial speed of cart 1, 1 point
or trials involving at least three different initial speeds
Example response:
Quantity to be Measured Symbol Equipment for Measurement
Initial speed of heavy cart (H) viH Motion sensor (MS1)
Final speed of heavy cart vf H Motion sensor (MS1)
Mass of heavy cart mH Triple beam balance
Final speed of light cart (L) vf L Motion sensor (MS2)
Mass of light cart mL Triple beam balance

MS1 H L MS2

Measure the mass of each cart with the triple beam balance. Set the carts and motion
sensors on the track. With the motion sensors recording, push the heavy cart toward the
light cart so that they collide. Record the motion sensors readings for the speeds of the
heavy cart immediately before the collision, and the speeds of both carts immediately
after the collision. Repeat 8 times, varying the force with which the heavy cart is pushed
so that a wide range of initial velocities for the heavy cart are used.

Total for part (b) 5 points


(c) For a correct indication of how initial and final total kinetic energy would be calculated 1 point
from the raw data

For using the difference between initial and final total kinetic energy to determine the 1 point
kinetic energy lost

For a description of how the calculated fractional energy losses would be used to confirm 1 point
or disconfirm the hypothesis

Example response:
For each trial, calculate the initial kinetic energy of the heavy cart using
Kinit = (1 2)mH viH 2 .
energy K f
Also calculate the final kinetic= (1 2)mH vf H 2 + (1 2)mL vf L 2 .

Make a graph of K lost =


K init ( K init − K f ) K init as a function of viH .
If a positive trend is seen in the data, the hypothesis is confirmed.

Total for part (c) 3 points

© 2022 College Board


AP® Physics 1 2022 Scoring Guidelines

(d) For using momentum conservation to find v f in terms of vi , or the ratio of post- 1 point

collision to pre-collision speed

For a correct expression for final kinetic energy (or consistent with v f found from 1 point
conservation of momentum)
OR

a correct expression for the fractional energy loss in terms of m1 , m2 , the initial speed of
the heavy cart, and the final speed of the two-cart system

A correct expression for fraction of kinetic energy lost: 1 point


m2 m1
or 1 −
m1 + m2 m1 + m2

Scoring note: The above point is not earned if the answer includes any variables other
than m1 and m2

Total for part (d) 3 points

Total for question 2 12 points

© 2022 College Board


AP® Physics 1 2022 Scoring Guidelines

Question 3: Qualitative/Quantitative Translation 12 points

(a) For an arrow directed upwards labeled Tension (or FT ) 1 point

For an arrow directed downwards labeled Force of Gravity (or mg , or Fg ). 1 point

Scoring note: G or g are not acceptable labels for the force of gravity
For an arrow labeled tension that is larger than the arrow labeled Force of Gravity. 1 point
Scoring note: a maximum of two points can be earned if there are any extraneous
forces.
Example response for part (a)

Total for part (a) 3 points


(b) For a sketch showing a non-linear path 1 point
For a sketch that is concave up 1 point
For a justification that addresses that the box is moving at a constant speed in the 1 point
horizontal direction and accelerating in the vertical direction which leads to a concave
up parabola.

© 2022 College Board


AP® Physics 1 2022 Scoring Guidelines

Example response for part (b)

Since the box is moving at a constant speed in the horizontal direction and accelerating
in the vertical direction, the path that the box takes should look like a concave up
parabola
Total for part (b) 3 points
(c) For a kinematic equation correctly relating the horizontal position of the box to the 1 point
horizontal component of the speed of the box and the time
x
vx =
t
x
0.5 m/s =
t
x
t=
0.5 m/s
For a kinematic equation correctly relating the vertical position of the box as a function 1 point
of the vertical acceleration of the box and the time
1 2
y= ayt
2
1
y = (1.0 m/s 2 ) t 2
2
For correctly combining the vertical and horizontal motions of the box into a single 1 point
equation.
2
1  x 
y = (1.0 m/s 2 )  
2  0.5 m/s 
y = ( 2 m −1 ) x 2

Total for part (c) 3 points

© 2022 College Board


AP® Physics 1 2022 Scoring Guidelines

(d) For an attempt to use functional dependence to describe how the equation and the sketch 1 point
in part B relate

For using functional dependence correctly to relate the equation derived in part (c) 1 point
consistently to the graph sketched in part (b).
Example response for part (d)
The equation I derived shows that the vertical position of the box is proportional to the
horizontal position squared and so the shape of the path traveled by the box should be a
parabola, as sketched in part (b).
Total for part (d) 2 points
(e) One point for an arrow pointing upwards and to the right. 1 point

Example response for part (e)

Total for part (e) 1 point

Total for Question 3 12 Points

© 2022 College Board


AP® Physics 1 2022 Scoring Guidelines

Question 4: Paragraph-Length Response 7 points

(a) For a labeled friction force vector in the correct direction in both diagrams 1 point

For labeled gravitational and normal forces in the correct directions in both diagrams, 1 point
with no extraneous forces
Total for part (a) 2 points

(b) Correct Answer: tdown > tup


Scoring note: The response is graded even if an incorrect selection is made.
For stating that the magnitude of the net force on the block is greater when it is sliding 1 point
up the ramp than when it is sliding down the ramp because the direction of the frictional
force changes while the direction of the component of the gravitational force along the
ramp does not (this can be implied) OR a description of the net force consistent with the
free-body diagrams (FBDs) in part (a)

For stating that the magnitude of acceleration of the block while sliding up the ramp is 1 point
greater than that when sliding down, OR a description of acceleration consistent with the
FBDs in part (a)

For a justification that v f is less than v0 , OR average v up is greater than average v 1 point
down (e.g., speed changes more on way up than on way down because acceleration is
greater on the way up and the same distance covered and final/initial speed on way
up/down is zero), OR a description of final and initial speeds consistent with the FBDs
in part (a)

For a correct argument that, if v f is less than v0 or average speed up is greater than 1 point
average speed down, then tdown is greater than tup . (This argument could include a
kinematic equation.)
Scoring note: Student cannot earn this point even if justification is consistent with
incorrect part (a) FBD

For a logical, relevant, and internally consistent argument that addresses the required 1 point
argument or question asked, and follows the guidelines described in the published
requirements for the paragraph-length response
Scoring note: A maximum of 4 of 5 points (first 3 points plus 5th point) can be earned
if the FBD in part (a) is incorrect (e.g., friction force is in the same direction in both
FBDs)

© 2022 College Board


AP® Physics 1 2022 Scoring Guidelines

Alternate solution using work/energy reasoning for 1st two points of part (b)

For correctly applying work-energy to up/down ramp (or, indicating block-Earth system 1 point
has the same potential energy at the beginning and end)

For correct treatment of energy loss to friction 1 point

Total for part (b) 5 points

Total for Question 4 7 Points

© 2022 College Board


AP® Physics 1 2022 Scoring Guidelines

Question 5: Short Answer 7 points

(a) For correctly relating the difference in maximum height of the ball to the kinetic energy 1 point
of the ball before and after the collision.

Example response for part (a)


If the collision between the ball and the floor were elastic, the speed just before the ball
hits would be equal to the speed of the ball just after it leaves the floor. Because of
conservation of energy, since air resistance is negligible, if the speed before and after the
collision were the same the ball should return to its starting height. Because it returns to
a lower height, we must conclude that there was energy lost in the collision, making the
collision inelastic.

Total for part (a) 1 point

(b) For using conservation of energy or work energy principle to attempt to find the speed of 1 point
the ball just before the ball hits the force sensor.

For using conservation of energy or work energy principle to attempt to find the speed of 1 point
the ball just after the ball hits the force sensor.

For correctly addressing the vector nature of momentum (or velocity) 1 point
Scoring note: Responses do not have to correctly calculate the change in momentum. A
response must show that the momentum (or speeds) have different directions, and that the
difference in these two are the algebraic sum of the two magnitudes.

Example response for part (b) p mv f − mvi


∆=
KE = U g ∆p ( 0.50 kg ) ( 4.7
= m
s ) − ( 0.50 kg ) ( −5.5 m
s
1 mv 2 = mgh
2 i ∆p =5.1 Ns
=vi 2 gh
= (
2 10 m
s2
) (1.5=
m) 5.5 m
s
U g = KE
mgh = 1
2
mv 2f

vf
= 2 gh
= (
2 10 m
s2
) (1.1=
m) 4.7 m
s

Total for part (b) 3 points

© 2022 College Board


AP® Physics 1 2022 Scoring Guidelines

(c) For attempting to calculate the area under the curve 1 point
For comparing the area under the curve to the value of the change in momentum for 1 point
part (b).

For a correct justification that the impulse shown from the force sensor is not large 1 point
enough to cause the ball to return to a height of 1.1 meters.

Example response for part (c)


The area under the force as a function of time graph is equal to the impulse or change in
momentum of the ball.

5.1 Ns
∆pobserved =
1 0.2 ms )( 4 kN ) + ( 4 kN )( 0.6 ms ) + 12 ( 0.2 ms )( 4 kN )
precorded
∆= 2
(
3.2 Ns
∆precorded =

The change in momentum recorded by the force sensor is significantly smaller than the
change in momentum determined from experimental data collected from the initial and
final heights of the ball. It is likely that the force sensor reading cannot be trusted.
Total for part (c) 3 points
Total for question 5 7 points

© 2022 College Board


AP Physics 1 - 2022 Practice Exam 2 Scoring Worksheet

Section I: Multiple Choice When an AP Exam is


administered, psychometric
× .8000 = analysis determines the score
Number Correct Weighted Section I Score ranges corresponding with
(out of 50) (Do not round)
each AP Exam score (5, 4, 3,
2, and 1) based on a
Section II: Free Response composite score scale that
combines and weights the
Question 1 × 0.8888 = exam parts. Due to minor
(out of 7) variations in exam difficulty,
(Do not round)
the number of points
Question 2 × 0.8888 = corresponding with each AP
(out of 12) (Do not round) Exam score can vary on
different exams. Because this
Question 3 × 0.8888 = practice exam was never
(out of 12) (Do not round) administered, AP has
developed these estimated
Question 4 × 0.8888 = score ranges that teachers can
(out of 7) (Do not round) use to approximate AP Exam
scores. We caution that these
Question 5 × 0.8888 = ranges, and the resulting AP
(out of 7) (Do not round) Exam scores, are only
estimates, and student
performance on this practice
Sum = exam does not necessarily
Weighted predict performance on a
Section II Score different exam.
(Do not round)

Composite Score

+ =
Weighted Weighted Composite Score
Section I Score Section II Score (Round to nearest
whole number)

AP Score Conversion Chart


Physics 1
Composite
Score Range AP Score
61-80 5
48-60 4
37-47 3
24-36 2
0-23 1
AP® Physics 1 2022 Question Descriptors and Performance Data

Multiple-Choice Questions

Question Science Practice Learning Objective Topic Key


1 1.4 3.A.3.1 2.5 C
2 2.2 5.D.2.5 5.4 B
3.A.1.1
3 5.1 1.1 B
3.A.1.3
4 5.1 3.A.1.1 1.1 D
5 5.1 3.B.3.3 6.1 D
6 2.2 3.B.2.1 2.6 C
7 2.2 3.B.1.3 2.6 A
8 2.2 4.C.2.2 4.2 C
9 5.1 4.B.2.2 5.2 B
10 1.4 3.B.2.1 2.6 C
11 6.4 5.B.4.1 4.3 B
12 6.4 5.B.5.4 4.3 C
13 6.4 3.E.1.1 4.2 C
5.A.2.1
14 6.4 4.1 D
5.B.4.2
15 1.4 3.B.2.1 3.7 B
3.A.1.1
16 1.4 3.B.1.3 2.6 D
3.B.2.1
17 1.4 5.B.4.2 4.3 C
3.E.1.1
4.C.1.2
18 7.2 4.2 A
5.B.3.2
5.B.5.5
4.C.1.1
4.C.1.2
19 1.4 4.2 D
5.B.3.2
5.B.5.5
20 1.5 3.A.1.1 1.1 A
3.E.1.1
3.E.1.4
21 1.4 4.C.1.1 4.2 A
4.C.2.2
5.B.4.2
3.A.1.1
22 2.2 4.3 C
5.B.5.5
3.B.1.1
23 1.4 3.B.2.1 2.6 C
4.A.2.1
24 7.2 3.A.3.1 2.5 C
25 6.4 5.B.5.4 4.3 B
26 7.2 5.D.1.1 5.4 A
3.F.2.1
27 1.4 7.3 C
4.D.1.1
3.F.2.1
28 2.2 7.3 D
4.D.3.1
29 6.4 3.F.2.1 7.2 A
30 7.2 3.B.3.1 6.1 C
31 6.4 4.A.2.1 1.2 C
32 6.4 4.A.2.2 1.2 A
3.A.1.1
33 1.5 1.1 B
3.A.3.1
AP® Physics 1 2022 Question Descriptors and Performance Data

Question Science Practice Learning Objective Topic Key


4.A.1.1
4.A.2.1
34 2.2 4.A.3.2 5.4 D
5.D.2.5
5.D.3.1
35 6.4 3.F.2.1 7.2 A
36 6.4 5.B.5.3 4.3 C
37 1.4 4.A.1.1 1.2 A
38 7.2 5.E.1.1 7.4 A
39 2.2 3.E.1.4 4.2 C
40 6.4 4.A.2.1 1.2 A
41 4.2 3.D.2.4 5.1 A
42 1.4 3.F.1.5 7.2 B
43 1.4 4.B.1.1 5.2 A
44 6.4 3.A.3.1 3.8 D
45 6.4 5.E.1.1 7.4 A

Question Science Practice Learning Objective Topic Key


3.A.4.3
131 1.4 2.6 A, C
3.B.2.1
3.D.2.2
132 5.1 5.1 B, D
3.D.2.3
133 2.2 3.A.1.1 1.1 A, D
2.B.1.1
134 7.2 3.4 B, D
2.B.2.1
135 6.4 4.C.1.2 4.2 B, D

Free-Response Questions

Question Science Practice Learning Objective Topic

1 1.4|1.4|1.1|1.1|1.1|2.2|2.2 4.A.2.3|4.A.2.3|3.B.2.1|3.B.2.1|3.B.2.1|4.A.2.3|4.A.2.3| 1.2|1.2|2.6|2.6|2.6|1.2|1.2

6.4|4.2|4.2|4.2|4.2|4.2|2.1|5.1| 5.D.1.4|3.A.1.2|5.D.1.4|5.D.1.4|5.D.1.4|5.D.1.4|4.C.1.1| 5.4|1.1|5.4|5.4|5.4|5.4|


2
5.1|2.2|2.2|2.2 5.D.1.4|5.D.1.4|5.D.2.5|5.B.4.2|4.C.1.1 4.2|5.4|5.4|5.4|4.3|4.2

1.1|1.1|1.1|7.2|7.2|6.4|2.2|2.2| 3.A.2.1|3.A.2.1|3.A.2.1|3.B.1.1|3.B.1.1|3.B.1.1|3.A.1.1| 2.5|2.5|2.5|2.6|2.6|2.6|


3
2.2|2.2|6.4|1.1 3.A.1.1|3.A.1.1|4.A.2.3|4.A.2.1|3.A.2.1 1.1|1.1|1.1|1.2|1.2|2.5

4 1.1|1.1|1.4|6.4|2.2|1.5|1.5 3.B.2.1|3.B.2.1|3.B.2.1|3.B.1.1|3.A.1.1|3.A.1.1|3.A.1.1 2.6|2.6|2.6|2.6|1.1|1.1|1.1

5 6.4|6.4|2.2|7.2|2.2|5.1|5.1 5.D.2.1|5.D.2.1|5.B.4.2|5.B.4.1|5.D.1.5|4.B.1.2|3.D.2.3 5.4|5.4|4.3|4.3|5.4|5.2|5.2

You might also like